Sie sind auf Seite 1von 159

HUNG VUONG SUMMER C AMP-2010

HUNG VUONG SUMMER C AMP-2010

TABLE OF CONTENT

FOREWORD PART I. RESOURCEFUL IDEAS FOR CLASSROOM INSTRUCTION Writing an introduction for an essay Do Thi Minh Hong Helping students with academic writing styles Le Thanh Thuy Link verb Le Thi Van Khanh Teaching essay writing to 10th graders Tran Thi Thu Ha The importance of material adaption Le Ngoc Oanh English intonation for different types of questions

2 5 6

15

21

31

43

Strategies for english language learners in the classroom Duong Minh Khai PART II. CLASSROOM PRACTICE RESOURCES Test 1 Test 2 Test 3 Test 4 Test 5 Test 6 Test 7 Test 8 Test 9 Test 10 PART III. KEY TO CLASSROOM PRACTICE RESOURCES

50

53 54 62 73 81 89 101 108 118 129 137 143

HUNG VUONG SUMMER C AMP-2010

FOREWORD

Hung Vuong Summer Camp, which has been in operation for several years, is an initiative of Professor Nguyen Van Mau, former President of Hanoi University of Science, with a view to motivating the students, and to creating an opportunity for them to acquire new knowledge and skills in their majors. Also, the Camp is a forum for teachers to learn from each other and to take in new professional input from academic advisers, who are university professors. The Camp involves 15 specialising upper secondary schools from the northern mountainous and highland provinces. English has just been included since 2009. This document is composed of three parts. Part I, Resourceful Ideas for Classroom Instruction, is a collection of essays written by teachers. Although the essays are not completely based on teachers reflections on their teaching, they provide useful information on how to deal with thorny issues in English language teaching, especially the issue of helping upper secondary school students improve their academic writing skills. The purpose of this part is two-fold. Firstly, it encourages teachers to enrich their teaching repertoire by reading the literature to keep themselves abreast of the latest theoretical developments in the field. Secondly, it helps teachers working in underresourced conditions to access innovative ideas for their professional development. In fact, these essays are resources for teachers to refer to in addressing the challenges they are likely to encounter in their classroom teaching. We hope that teachers will work on these new ideas, then reflect on their practical experiences in order to develop more context-sensitive instructional practices. After all, one of the pathways to professional development is looking at theories through the lens of practical experience and reflecting on the experience in light of theories. My belief is that nothing is more practical than a good theory, and practice without theory would become routinised. Part II provides resources for classroom use. They are presented in the format of a test, contributed, again, by teachers from different schools. It is worth noting that these tests do not follow a consistent format for the reason that the need of diverse classroom input has to be satisfied. These resources are to save time of busy teachers developing their own materials to prepare their students for different examinations. Therefore, its up

HUNG VUONG SUMMER C AMP-2010

to teachers to cut and paste which parts of these tests they think are appropriate to their own students and their own purposes. It is also our purpose that the students can use these resources for their self-study. Therefore, the key answers to the tests are provided in Part III of the document. Since the purpose of Hung Vuong Summer Camp is to make a contribution to the improved quality of both teaching and learning particular subjects, we hope that teachers and students will find the document useful in some way. Needless to say, there remains space for improvements in terms of the input and the format of the document. We therefore appreciate more thoughtful contributions to the following volumes.

Academic Adviser

LE VAN CANH, PhD

HUNG VUONG SUMMER C AMP-2010

PART I

RESOURCEFUL IDEAS FOR CLASSROOM INSTRUCTION

HUNG VUONG SUMMER C AMP-2010

WRITING AN INTRODUCTION FOR AN ESSAY

Do Thi Minh Hong Bac Giang Specializing Upper Secondary School

Writing, in general, is always considered the most difficult skill of the four skills. Not only students but teachers as well find it hard to master. How to write a good essa y, especially an essay for academic purpose, is a challenging skill that not many students can successfully acquire. Though students have been helped with the knowledge that this skill requires their textual competence, their ability to organize and develop the ideas in a persuasive way and their cultural and academic backgrounds of the language they are learning; though they have tried hard, they find writing, especially academic writing, a very difficult task to overcome. This paper does not deal with writing academic essays in general, but suggests the methods of writing an introduction for an argumentative essay only, the most popular type of essays that students often encounter when taking The National Exam every year. Because we all believe that writing a good introduction will help to lead to a good essay in the end. As we all know, when students are asked to write an essay, the first step they have to do is to plan their writing. After this step, it is suggested that students spend several minutes thinking carefully about what they are going to say before writing. Because if they make a good start, it will be easier for them to keep their thought around the key elements of the topic smoothly through to the end. The purpose of the introduction is to introduce the topic to the audience and make him clear about what we are going to write about. There are many ways to write an introduction and none of them are wrong. The introduction for an argumentative essay is different from that of others. An argumentative essay is usually a discussion of a topic, giving reasons why the writer holds a particular point of view. Most academic papers or articles are laid out in such a way that the introduction sets out the problem, and the paragraphs that follow give supporting arguments for the writers point of view. If we look carefully at most professionally written academic articles, we will find

HUNG VUONG SUMMER C AMP-2010

that the introduction is laid out more or less like this: 1. 2. 3. Giving necessary background information about the topic; Indicating the issue the writer plans to develop in the body of the essay; Concluding with a statement that sets out the writers own opinion on the topic.

However, it is not quite easy to do this well. The fact is that, Vietnamese students prefer thinking in their own language, their mother tongue I mean, to thinking in the language they are learning, a foreign language. What they do when being asked to write an essay is try to think and write as long as they can with a lot of ideas around the topic, regardless they are really needed or not for the essay. This is a habit of Vietnamese students that needs a lot of time to change. In many cases, they tend to give details in the introduction or sometimes, even they give something that is far from the topic or something that has no relation with the topic they are going to write about. To better the situation, I think, teachers should make everything clear enough to help students when teaching this skill. To make the three points Ive mentioned above clear and easy for students to apply, we are going to, step by step, deal with each of them.

1.

Giving necessary background information about the topic. The general statement with which we begin our introduction should make our

audience interested in our topic. As an essay that starts by arousing the tutors interest is likely to gain more marks than one which doesnt. Giving necessary background information about the topic is a good way of getting to the point. Often, we will need to write several sentences to describe the topic, but sometimes only one sentence would be enough. Example: 1. 2. The problems of air pollution have been widely reported in the worlds press. It has been said in a report that many people are victims of calcuholism, a

dependence on the issue of calculators, causing a diminished ability to do mathematics on ones own.

2.

Indicating the issue the write r plans to develop in the body of the essay.

HUNG VUONG SUMMER C AMP-2010

The issue defines what the essay is going to be about. This is perhaps the most important part in the introduction, because : 1.by reading the question, the audience will know from the beginning what the essay is going to be about. 2. putting down the question helps to clarify the writers own mind and 3. it helps the writer to stick to one main point in the essay. Example: Topic: School Leaving Age Introduction: (1) Recently, the school leaving age was raised to fifteen years. (2) Is it really a right way to benefit our society? (3)I think all young people should stay at school until they are eighteen years old, not fifteen. Concluding with a statement that sets out the writers own opinion on the The statement of the writers own opinion on the topic can be called the main idea statement. This statement will tell the readers what we think about the issue, and it is the answer to the question posed by the issue. Note that we do not offer any reasons why you hold this opinion in the introduction and we will give reasons in the body of the essay. Example: Topic: A home for Young Foreign Students Introduction: (1) Many people find it advantageous to purchase a home, but others find renting more suited to their needs. (2) Which might be better for young foreign students? 93) Although there are advantages for both options, renting is generally a better choice. In short, to have a good essay with main points solved satisfactorily, the writers have to know well what they have to do in each section/ part of the essay. The introduction is thought to be the most important part of an essay that helps the writer to access their po int. A good introduction will help them to write a good essay and receive a good score.

3. topic.

HUNG VUONG SUMMER C AMP-2010

HELPING STUDENTS WITH ACADEMIC WRITING STYLES

Le Thanh Thuy Hoang Van Thu Specializing Yppe r Secondary School, Hoa Binh

1. Introduction The art of argumentative writing is not an easy skill to acquire for students learning English as a foreign language and it tends to get rather neglected in many classes. However, argumentative writing seems to be one of the most important, yet difficult, academic skills for gifted students in many specialized schools all over the country. In our previous article, although some common causes of the lack of interest in argumentative writing have been addressed and solutions have been suggested, it could not fully satisfy the readers whose needs focus on teaching writing professionally and effectively. That is also the main reason why the writer wishes to, once again, consider argumentative writing the targeted matter to study. However, in this article, the writers intention emphasizes on one aspect of academic writing, that is academic writing style. Definition of academic style and some strategies to help secondary students be familiar with academic writing style are included in this paper.

2. What is academic style? Writing style, in general, is the manner in which a writer addresses a matter. When academic style is mentioned, the term is used to refer to a particular style of expression.

HUNG VUONG SUMMER C AMP-2010

The overall structure of academic writing is formal and logical. In other words, academic writing is linear, that is, it has one central point or theme with every part contributing to the main line of argument. There are five main features of academic writing that are often discussed: complex, formal, objective, explicit and hedged. With regard to the complexity, academic writing should possesses some of the characteristics. First, it should be lexically more dense and has a more varied vocabulary, which means more noun phrases and less verb phrases. Second, the written text are shorter and the language has more grammatical complexity. That is more subordinate clauses and more passive are used. Formality is the second feature which should be taken into consideration. It is relatively simple in the way that the writer should avoid using colloquial words as well as abbreviated ones. Objectivity concerns much about the impersonal information and argument that are conveyed in the piece of writing while explicitness focuses on how information and ideas are communicated. Academic writing tends to use nouns and adjective rather than verbs and adverbs and system of signaling words is exploited to the highest of efficiency. Another typical feature of academic style is hedging. It is often believed that academic writing is factual, simply to convey facts and information, however, the concept of cautious language now is considered the safe and wise way to express the writers opinion on a subject matter. Some introductory verbs, modal verbs and that clauses may work well in this case. ( Introductory verbs: seem, tend, appear, believe, doubt, suggest...; modal verbs: will, would, may, might, That clauses: It could be the case that, It might be suggested that, It is often claimed that ....) One example of the difference between academic style and normal style is presented as follows: Normal Because the technology has improved its less risky than it used to be when you install them at the same time, and it doesnt cost so much either. Academic Improvements in technology have reduced the risk and high cost associated with simultaneous installation.

10

HUNG VUONG SUMMER C AMP-2010

As the definition of academic writing style has been provided, the next part of the article will propose some effective strategies with a view to helping students be familiar with academic writing style.

3. Some strategies should be adopted when teaching acade mic writing. 3.1. Studying sample writings to be accustomed to academic English styles It is advisable that students refer to authentic argumentative writings so as to gain clear perceptions in the executions of introducing a topic, stating an opinion, presenting arguments and examples, justifying their own viewpoint ... Furthermore, students can learn many things about grammatical structures, vocabulary, format and lay - out of an argumentative writing by reading and analyzing authentic samples. By doing so, the differences between writing styles can be gradually settled. The students study the samples then attempt to apply some useful techniques in their own argumentative writings. 3.2 Assigning exercises on distinguishing between debatable and non debatable topics. An argumentative essay is built around a specific statement (or main premise) that is debatable within the field in which you are studying. In other words, at the centre of an argumentative essay is a statement with which your readers may disagree. These are sometimes called arguments, assertions, propositions or premises. Non debatable, on the other hand, are statements with which no-one would normally disagree or argue. These are sometimes called "facts". To make students gain clear perception of this, teachers can assign task in which student have to consider to come up to the decision whether this statement are debatable or non debatable. A sample task is supplied as follows: Which of the following statements are debatable and which are non-debatable? If the statement is debatable, put a tick in the box next to the word "debatable". If the statement is non-debatable, put a tick in the box next to the word "non-debatable". 1. Computers and automation increase unemployment debatable non-debatable

11

HUNG VUONG SUMMER C AMP-2010

2.

Smoking is harmful to people's health debatable non-debatable

3.

Plants produce oxygen that the world needs to sustain life debatable non-debatable 4. Australia has some of the most venomous snakes in the world debatable non-debatable

5.

A good education is necessary for a successful and happy life debatable non-debatable

By deeply understanding the kinds of topic, students will find it easier to come up with arguments for and against it. Moreover, when they choose a topic to practice themselves, this skill is essential and practical. 3.3 Using however as a powerful tool to provoke an argument Using connectives is by no means the effective way to make the writing clear and easy to follow. There are some types of connectives such as: listing, information adding, contrasting ... However, the gold word that the writer wishes to emphasize is however. Why this word? As critical thinking is clearly conveyed through academic writing, showing the writer is well aware of both sides of the issue is very essential. Thus, using however, to shift from an opposing argument to a supporting one is one the most effective way for the writer to achieve his/her purpose. Teachers should assign students with opposing arguments or ideas and require their students build up a hedging statement and using however to present the other side of the problem. Some typical phrases of

12

HUNG VUONG SUMMER C AMP-2010

problematising should also be supplied to make students find it easier dealing with this kind of task. Some hedging phrases are: It may be/ might be/ could be/is/ has been argued/ asserted/ contended/ maintained/ claimed/ said/ that .................. . However, ................... Sample task There are 6 issues listed below. Your task is to write a pair of sentences similar to the sentence in the model about each of the issues. The first sente nce should present and problematise the opposing opinion on this issue. The second sentence should present your opinion (or premise) on this issue, Dont forget to link the two sentences with the contrasting connective, However. Use the language summary above to help you and try different ways of showing you are aware of the opposing opinion. 1. 3. Banning smoking in restaurants Making Australia a republic 2. 4. Banning all guns Decreasing immigration levels in Australia 5. Having the Grand Prix at Albert Park 6. Reintroducing capital punishment

Sample answe r 1. Banning smoking in restaurants It could be asserted that smokers should be free to smoke in restaurants as they wish. However, by banning smoking in all public places we can help people to give up this extremely unhealthy habit. 2. Banning all guns It has been argued that people have a legal right to own guns in our society. However, unlike the United States, there is no law in Australia that guarantees any right to own firearms. 3. Making Australia a re public It is claimed that changing to a republic would make Australia a more democratic country.

13

HUNG VUONG SUMMER C AMP-2010

However, the constitutional system that we have at the moment has functioned very well to ensure democracy for nearly 100 years and it is very risky to change it. 4. Decreasing immigration levels. It has been maintained that there are not enough employment opportunities in Australia to sustain high immigration levels. However, research into this issue has found that immigrants create new forms of employment through the businesses they establish. 5. Having the Grand Prix at Albert Park. It is contended that the people of Melbourne are very unhappy with the government's decision to stage the Grand Prix at Albert Park. However, recent opinion polls show that the vast majority of the population is not concerned with this issue at all and is happy to have the Grand Prix anywhere in Melbourne. 6. Reintroducing capital punishme nt It could be said that reintroducing capital punishment will deter people from committing serious crimes of violence. However, overseas experience indicates that the level of violent crime is not reduced by the threat of the death penalty

14

HUNG VUONG SUMMER C AMP-2010

LINK

Le Thi Van Khanh Thai Nguyen Specialing Upper Secondary School

During the years of teaching English at the upper secondary schools, I have found that every part of grammar is very interesting if one is devoted to studying it. When studying the forms of verbs and their functions in English, I want to deal with the problem Linking verbs- a most difficult subject for the learners and those who study English. 1. Link verbs: Link verbs have partly lost their lexical meaning and are used as part of a compound nominal predicate. The main lexical meaning of this kind of predica te is expressed by the predicative ( a noun, an adjective, or a verbal).

Link verb + Adj Eg: - She is good. - She becomes more beautiful. - He s getting tired now. 2. Struture:

15

HUNG VUONG SUMMER C AMP-2010

Link

Link verbs + Predicatives (N, ADJ, G, PI or PII)

Eg: - She is a student. (noun) - My sister looks beautiful. (adjective) - He gets tired. (participle2 ) - It is surprising that he passed the exam. (participle1 ) - Her hobby is collecting stamps. (gerund) 2. Groups of linking verbs: With regard to their meaning, linking verbs fall under three groups: * Verbs of being: be, look, feel, sound, smel, taste, fall, etc.. * Verbs of becoming: become, come, get, turn, grow, go, run, etc * Verbs of remaining: remain, keep, stay, seem, appear, etc. Eg: - He usually feels tired after working hard. - His mother went mad when her son didnt obey her. - After a long time when I met her, she still remained beautiful. When used as linking verbs, they almost lose their lexical meaning and are to be followed by an adjective, not by an adverb. Eg: - It sounds quite correct. (not corectly) - She looks very nice. (not nicely) - The apple tastes bitter. (not bitterly) ** Notes: Differences between Linking verbs & notional verbs Linking verbs - He goes mad. - She remains tired. - I get used to stay up late. - The water runs deep. Her face turns pale. Notional verbs - We go to work everyday. - Ill remain with you. - She never gets up late. - He is running across the road. - He turned round quickly.

3. Some types of words after linking verbs: 1, Nouns or noun-phrases - She is a beauty. - She is a nice girl. 2, Pronouns

16

HUNG VUONG SUMMER C AMP-2010

- Its me that you always help. - The book is mine. 3, Adjectives or compound adjectives - She seems more beauiful. - She is oval- faced. 4, A numeral - Ill be 40 next week. - Ladys first. 5, An infinitive or an infinitive construction - My plan is to start soon. - The best thing for you is to join us. 6, A gerund or a gerundial construction - My hobby is fishing. - This is not playing the game. 7, A participle - He looked embarrassed. - This film is exciting. 8, A clause - It seems that he is sleeping. - The best thing to do is what your advisor suggested.

3.. Linking ve rbs accompanied by adjectives

Linking ve rbs

Adjectives

Linking ve rbs

Adjectives

17

HUNG VUONG SUMMER C AMP-2010

be become come fall feel get go grow keep make prove seem

young / old more beautiful true ill / asleep tired / sleepy exhausted / bored mad / wild prosperous / tall quiet / silent clear / clean right / wrong hot / cold / wet

stay remain smell taste sound turn take run sit stand hold appear

handsome young delicious good interesting red / pale ill deep / strong still / tight / pretty still / empty / ready still / good much colder

C. Exercises: 1. Exercise of multiple choice I dont think John looks well today

Fill in the blanks with s uitable words by circling A, B, C, or D. 1. He seems _______ a. fairly tired b. fair tired c. fair tiredly d. fairly tiredly 2. Which do you prefer, the blue china or the white china? The white china is ______ a. definitely better b. better definite c. betterly definite d. better definitely

3. I dont know she seemed ______ in it, however a. interest b. interesting c. interested d. interestingly 4. This hand-made cloth is beautiful To see such quality is ______ a. pleasing b. pleased c. pleasant d. pleasure

5. I think Mary looks ______ today. a. good b. badly c. well d. happily

6. The crowd grew ______ as the official began her speech. a. quiet b. quietly c. noisily d. silently

7. Oh! This dish smells ______.

18

HUNG VUONG SUMMER C AMP-2010

a. well

b. delicious

c. deliciously

d. wonder

8. John felt ______ after lunch. a. asleep b. sleepy c. sleepless d. sleep

9. It appears ______ that Mary Hanson will win the election. a. certainly b. certain c. sure d. surely

10. The weather turns ______ in July. a. hot b. heat c. hottest d. cold

**Key: 1. a 2. a 3. c 4. a 5. c

6. a

7. b

8. b

9. b

10. a

2. Exercise of word-form Find out the correct words to fill in the blanks, using the words in the brackets. 1. 2. 3. 4. 5. 6. 7. 8. 9. 10. He is ______ (die) The contract offer sounded ______ to me, so I accepted the job. (fairly) It is becoming ______ to go out alone at night. (danger) Divorce is getting more ______. (common) The radio has gone ______. (wrongly) She turns ______ with cold. (blueness) Has she fallen ______? (illness) The water ran ______ when I turned the tap on. (coldness) For many years, he has stayed ______. (single) The task proved ______ than wed thought. (difficulty)

**Key: 1. dead 6. blue 7. ill

2. fair 8. cold

3. dangerous

4. common

5. wrong

9. single

10. difficult

3. Exercise of rewriting Rewrite these sentences , using bold words. 1. 2. 3. He went mad when I told him that Id changed my mind. (angry) I was really annoyed about missing that train. (annoying) It is exciting to travel all over the world. (exciting)

19

HUNG VUONG SUMMER C AMP-2010

4. 5. 6. 7. 8. 9. 10.

She looks sad. (seems) Look! Hes sleeping. (asleep) It is certain that she will come back. (appears) When the wind started to blow, I grew anxious. (got) The milk tasted sour. (turned) please, keep still while I take your photo. (sit) Are you single? Yes, I am. (stay)

** Key: 1. 2. 3. 4. 5. 6. 7. 8. 9. 10. He felt angry when I told him that Id changed my mind. Missing that train was annoying to me. Travelling all over the world is exciting. It seems that she is sad. / She seems to be sad. Hes falling asleep. It appears certain that she will come back. I got anxious when the wind started to blow. The milk turned sour. Please, sit still while I take your photo. Do you stay single?

20

HUNG VUONG SUMMER C AMP-2010

TEACHING ESSAY WRITING TO 10th GRADERS

Tran Thi Thu Ha Ha Long Specializing Upper Secondary School, Quang Ninh

I. Introduction Writing, along with other skills, is very important when we want to evaluate the English level. Many students, as well as teachers, consider writing so difficult that they hardly master it. However, the students are not only to blame for this problem because teachers' quality of English, lack of materials and the old conception in teaching and learning are also the main reasons. This is the reason why I would like to share some techniques I often use in teaching writing an essay for 10th form students of English. I hope that this will make a modest contribution to the enhancement of students writing performance.

Sentence Writing vs. Paragraph Writing Sentences and paragraphs are the most basic components of an essay. As a matter of fact, it is really necessary to study sentence structures, types of sentences and the basic rules when writing a paragraph when we want to learn how to write an essay. What follows is a review of various types of sentences and their characters in English. 1. Simple sentence: A clause is a group of words containing a subject and a verb. Some clauses can stand alone as a sentence. This kind of clause is an independent clause. A simple sentence is one independent clause. Eg: Tom loves Erica.

21

HUNG VUONG SUMMER C AMP-2010

2. Compound sentence: A compound sentence is two independent clauses joined together by a comma and one of the seven coordinating conjunctions: and, but, so, or, nor, for, yet. Example: Tom loves Erica, and she loves him Jack loves Jean, but she loves Ronald. Jack should forget Jean, or he will die a lonely man. Jean doesnt love Jack, so she wont marry him

or by semi colon and the first word after a semicolon is not capitalized. Example: Jack loves Erica; she loves him. Jack loves Jean; she loves Ronald.

or joining the two independent clauses with a semicolon + sentence connector + co mma: moreover, furthe rmore, however, otherwise, therefore. Example: Tom loves Erica; moreover, she loves him. Tome loves Erica; furthermore, she loves him.

3. Complex sentence: 3.1. Dependent clause: Dependent clause is a clause with a subject and a verb that does not express a complete thought and cannot stand alone. There are different kinds of dependent clauses: dependent adjective clauses : which can work twenty hours a day/ who cannot dependent adverb clause: before the company installed robots/ because.. dependent noun clause : that robots are here to stay

work around the clock.

3.2. Complex sentence: A dependent clause must be connected to an independent clause in order to make a complete sentence which is called a complex sentence. Independent clause dependent clause

22

HUNG VUONG SUMMER C AMP-2010

The company uses robot, which can work twenty four hours a day. Production was average before the company installed robots. Annual sales are now over one million because production meets buyers demands.

The word that begins a dependent clause is called a subordinating conjunction: which, who, before and because 3.3. Complex sentences with Adverb clauses Dependent adverb clauses tell why, when and where or introduce an opposite idea. to tell why: because, since as. to tell when and where: when, whenever, since, while, as soon as, after, before to introduce an opposite idea: although, though, even though

Here are some exercises for practice: Exercise 1: Combine each of the following pairs of sentences to make compound sentences. Use all the three ways you have just learned and punctuate carefully. 1. Robot can do boring, repetitive work. They can do unsafe jobs. 2. Robots can make minor decisions. They cannot really think. 3. Robots dont get tired, sick, or hungry. They can work twenty-four hours a day. 4. Human factory workers must learn new skills. They will be out of work because of robots. Exercise 2: Write independent or dependent in front of each of the following clauses. 1. When we arrived at the airport two hours later. 2. We arrived at the airport two hours later. 3. Because the teacher gave such hard exams. 4. After they got married. 5. Afterward, they got married. 6. Which is not an easy thing to do. 7. Which student got the highest grade. 8. Who will go to the store? 9. While I was a student in high school. 10. Where he parked his car.

23

HUNG VUONG SUMMER C AMP-2010

Exercise 3: Write complex sentences using the given subordinating conjunction (because) . ( although) . (before) . (when) .

II. Parts of a paragraph. A paragraph is a group of related sentences that develops one main idea, which is the topic of a paragraph. A paragraph is made up of three kinds of sentences that develop the writers main idea. These sentences are : the topic sentence supporting sentences the concluding sentence

1. The topic sentence: The topic sentence is the most general and important statement of the paragraph. It is t he key sentence because it names the subject and the controlling idea, opinion or feeling about the topic. The topic sentence should be written at the beginning of the paragraph because it tells the reader what you are going to say and you can look back at the topic sentence as you write the supporting sentences. The topic sentence must have the subject and the controlling idea. The controlling idea limits what you will write about in your paragraph. River rafting is challenging sport with important requirements S CI

24

HUNG VUONG SUMMER C AMP-2010

College-> Greenhill college-> registration-> frustrating experience Registration at Greenhill College is a frustrating experience

2. The supporting sentences: They develop the topic sentences by giving specific details , e xplanations, or proof about the topic sentence. Topic sentence: Traditional American family relationships have changed greatly in the last thirty years - x out of x marriages ending divorce - x out of x children live in homes with only one parent. - x percent of couples living together are not legally married. Topic sentence: Tokyo is the most expensive city in the world. - Cost of a diner at a medium-priced restaurant. - Rent for an average two-bedroom apartment. - Cost of a ride on public transportation - Cost of a medium-priced hotel room

3. The concluding sentence It signals the end of the paragraph. It summarizes the main points of the paragraph. It gives a final comment on the topic and leaves the reader with the most important ideas to think about. The concluding sentence reminds the readers of the topic sentence, it is like the topic sentence but in different words not copy the topic sentence. It can be begun with: All in all, In any event, In brief, Indeed, In other words, In short, Therefore, In conclusion, In summary, Finally Here are examples of topic sentences: - River rafting is a challenging sport with special requirements - Gold, a precious metal is prized for two important characteristics And here are the concluding sentences:

25

HUNG VUONG SUMMER C AMP-2010

- In short, if you are fearless and in good physical condition and can react quickly, river rafting is the ideal outdoor sport for you . - In conclusion, gold is treasured not only for its beauty but also for it utility. III. Some steps in writing a paragraph 1. Pre-writing: Brainstorming: Brainstorming is a prewriting activity in which you come up with a list of ideas about a topic on your own or in small groups with your classmates -Brainstorming may follow these steps: + Write down your general subject or specific topic. + Make list of everything that comes to your mind about it. + Use words, phrases and sentences + Keep writing down whatever comes up to your mind until run out of ideas.

2. Outlining a paragraph Simple outline: Topic sentence: Main supporting sentence Main supporting sentence Main supporting sentence

Concluding sentence Topic sentence A. 1. 2. 3. B. 1. 2. 3. C. 1. Main supporting sentence Supporting detail Supporting detail Supporting detail Main supporting sentence Supporting detail Supporting detail Supporting detail Main supporting sentence Supporting detail

26

HUNG VUONG SUMMER C AMP-2010

2. 3.

Supporting detail Supporting detail

Concluding sentence Example: Snow skiing Snow skiing must take extreme precautions on the slopes. A. 1. 2. 3. They must consider the weather conditions Temperature Wind Storm or clear weather

B. They must consider the slope condition 1. 2. 3. Surface Visibility Strength

C. They must consider their own ability 1. 2. 3. Beginner Intermediate Expert

Snow skiing is a safe and enjoyable winter sport if skiers take a few precautions

Here are some exercises for students to practice: Exercise 1: Study the following pairs of sentences and check the one you think would be a good, clear topic sentence for a paragraph 1. 2. 3. 4. 5. 6. 7. Snow skiing in the highest mountainsides requires great skill Snow skiing is fun Exercise is healthful Jogging is healthful for several reasons. Camping is a great outdoor activity. Camping requires a variety of special equipment. The legal age for drinking alcoholic drinks should be twenty-one for several

reasons.

27

HUNG VUONG SUMMER C AMP-2010

8. 9. 10.

Drinking is dangerous to your health. Small cars are popular. Driving a WV Rabbit saves money.

Exercise 2: Write topic sentences on the following topics. Limit the topic and the controlling idea. - Television - Sports - Internet - Computers - English - Smoking cigarettes Exercise 3: Writing supporting sentences 1. Smoking in restaurant should be prohibited for several reasons. a b c 2. International students have difficulties taking notes in class for several reasons.

a....................................................................... b c

IV. Writing an essay 1. What is an essay? An essay is a piece of writing several paragraphs long instead of one or two paragraphs. It is written about one topic. An essay has three main parts: 1. An introductory paragraph 2. A body ( two or more paragraphs) 3. A concluding paragraph The introductory paragraph consists of two parts: + general statements : - introduce the topic of the essay

28

HUNG VUONG SUMMER C AMP-2010

- give background information on the topic + thesis statements: - States the main topic

- Lists the subdivisions of the topic - May indicate the method of organization - The last sentence in the introductory paragraph Ex: A person born in the twentieth century has seen a lot of changes take place in almost all areas of human life. Some people are excited by the challenges that these changes offer; others wants to return to the simpler, less automates lifestyle of the past. Living in the twentieth century has certain advantages such as a higher standard of living but it also has some disadvantages such as a polluted environment, the depersonalization of human relationships, and the weakening of spiritual values The body consists of one or more paragraphs. Each paragraph develops a subdivision The concluding paragraph + a summary of the main points or restatement of your thesis in different words + your final comment on the subject, based on the information you have provided Ex: In conclusion, although the 20th century has indeed given us a lot of advantages by making us richer, healthier, and freer to enjoy our lives. It has, in my opinion, not made us wiser. The 20th century has also made our Earth dirtier, our people less humane and our spiritual lives poorer. We should continue to enjoy the benefits of technological advancements because they free us to pursue our interests and goals. However, we must make an effort to preserve our natural environment for future generations. Moreover, we should take the time now to make our lives more meaningful in an increasing impersonal world.

2. Some transitional signals used in writing paragraphs and essays Usage To list ideas in time order or order of importance To add another idea Sentence connectors Conjunction Coordinator Subordinator

First, second (ect..) First of all, then, next, after that, finally Furthermore, also, in addition, and , moreover, besides, what's more

29

HUNG VUONG SUMMER C AMP-2010

To add an opposite idea To add a similar idea To give an example To give a cause( or reason) To give an effect ( or result) To add conclusion

On the other hand, however.. Similarly, likewise, also For instance, for example

but and

Although, even though

for Therefore, thus, consequently, as a result In brief, all in all, indeed, in other words, in short, in the end. so

Because, since, as

Part C: Conclusion I. Conclusion Writing is important and necessary for student if they want to be fluent inon language in general and English in particular. Therefore, the teaching and learning of it must be focused on. In order for the students to learn how to write an essay well, teachers must teach them how to write individual sentences and individual paragraph. This paper is written with the hope that the skill of teaching writing of teachers and learning writing of students will be improved. It is hard to present more within the scope of this study, so the author hope that more aspects of writing such as describing graph will be studied by other teachers of English. Any comments and suggestions for this paper will be welcome and highly appreciated.

II. Suggestions for further study. Beyond the scope of my paper, I strongly suggest the following topic to further study. Some techniques in describing graphs and tables Some common mistakes of students when leaning writing Teaching sentence transformation

30

HUNG VUONG SUMMER C AMP-2010

THE IMPORTANCE OF MATERIAL ADAPTATION

Le Ngoc Oanh Son La Specializing Upper Secondary School

I. The proble m For many years, English teachers in high schools have been using their textbooks and teaching materials faithfully. However, in many cases, these materials do not reflect the reality of the classroom in terms of the students needs, wants and interests. Block (1991, p.211) states that despite the botaneous harvest of ELT materials which the past decade and a half provided, published materials do not always provide the types of texts and activities that a teacher is seeking for a given class. Many teachers might feel that something is missing in regards to materials and they do not know how and what to provide them. Materials Development and Adaptation has become a teaching topic that has caused great concern among English teachers as a way to bridge the gap between what English textbooks offer and what the students really need in their learning environment. In Vietnam in the recent years, the old textbooks series (Tieng Anh for 3-year students & English for 7- year students) have become out of date. The publication of the new series of textbooks has been welcomed by teachers and students throughout the country. However, it is undeniable that when applying this textbook series into teaching and learning, teachers and students in such a mountainous province as Son La encounter many difficulties. The problem is that many activities and exercises in the textbooks are not suitable to the reality of the classrooms in high schools in Son La Province and need to be

31

HUNG VUONG SUMMER C AMP-2010

adapted flexibly to be more effective. Nonetheless, not many English teachers here understand about Materials Adaptation and they are not provided a practical guide to textbook adaptation. In view of the preceeding discussion, this article attempts to provide the definition of Materials Adaptation, the characteristics required for adapted materials, the process of materials adaptation, the importance of layout. It is hoped that the study can be valuable to the English teachers in the province as the result of taking advantages of the study and its recommendations for a better strategies in adapting the textbooks in their teaching.

II. Definition of Materials Development and Adaptation

2.1. Materials: It is important to define materials as anything used by teachers and/ or students to facilitate the learning of a language. materials could obviously include cassettes, videos, CD-Roms, DVDs, dictionaries, grammar books, readers, workbooks, p hotocopied exercises, all kinds of realia, lectures and talks by guest speakers, Internet sources, and so on. (Tomlinson, 1998, p.2). Kitao (1998, p.1) also defines materials as the center of instruction and one of the most important influences on what go es on in the classroom. Obviously, the definition given by Kitao represents a great responsibility on the part of the teachers who must seriously consider what materials to use in their teaching.

2.2. Materials Development: Tomlinson ( 1998, p.2) refers Materials Development as anything which is done by writers, teachers or learners to provide sources of language input and to exploit those sources in ways which maximize the likelihood of intake, in other words the supplying of information about and/or experience of the language in ways designed to promote language learning. Ramirez Salas (2004, p.2) considers that Materials Development encompasses any kind of activity and/or exercise ( games, role plays, readings, problem-solving situations, group discussions, etc.) totally developed from raw texts, with or without pedagogical purposes, for the students level and created to address a section of the course content, that seems to

32

HUNG VUONG SUMMER C AMP-2010

be weak or lack further development or practice. Materials Development ranges from creating a short grammar exercise to writing a complete a textbook.

2.3. Materials Adaptation: Within the concept of Materials Development, there is a commonly used term that also needs to be defined: Materials Adaptation. According to Ramirez Salas (as cited in Maley, p.281), Materials Adaptation refers to the application of some strategies to make the textbook more effective and flexible. These strategies are omission, addition (adding extra material), reduction, extension (lengthening an activity to draw attention to other language features), rewriting/ modification, replacement, re-ordering, and branching (offering alternative ways to do the same activity).

2.4. Materials Development in contrast with Materials Adaptation: Contrasting with Materials Development, when teachers adapt an activity or exercise from a textbook, they use that activity or exercise as the basis to make certain changes. This is the process many teachers usually follow in their teaching, since they can use old textbooks to take texts and ideas and adapt them to their current teaching needs. Both processes are time-consuming and time-demanding. However, Materials Adaptation is easier in the sense that teachers can use texts given in different textbooks, whereas in Materials Development everything must be created from scratch.

III. Reasons to adapt materials Some teachers may question the need to adapt teaching materials if everything they need is already in a textbook elaborated by people who really know. being this a very valid opinion, it is necessary to analyze the advantages and disadvantages of using a textbook, and from there, discuss the need to adapt materials.

3.1. Advantages and disadvantages of using a textbook: Advantages: Richards (p.1) states the principal advantages of using textbooks: (i) They provide structure and a syllabus for a program.

33

HUNG VUONG SUMMER C AMP-2010

Without textbooks a program may have no central core and learners may not receive a syllabus that has been systematically planned and developed. (ii) They help standardize instruction. The use of a textbook in a program can ensure that the students in different classes receive similar content and therefore can be tested in the same way. (iii) They maintain quality. If a well developed textbook is used students are exposed to mate rials that have been tried and tested, that are based on sound learning principles, and that are paced appropriately. (iv) They provide a variety of learning resources. Textbooks are often accompanied by workbooks, CDs and cassettes, videos and comprehensive teaching guides, providing a rich and varied resource for teachers and learners. (v) They are efficient. They save teachers time, enabling teachers to devote time to teaching rather than materials production. (vi) They can provide effective language models and input. Textbooks can provide support for teachers whose first language is not English and who may not be able to generate accurate language input on their own. (vii) They can train teachers. If teachers have limited teaching experience, a textbook together with the teachers manual can serve as a medium of initial teacher training. (viii) They are visually appealing. Commercial textbooks usually have high standard of design and production and hence are appealing to learners and teachers.

Disadvantages: According to Richards (p.2), using textbooks has some potential negative effects as following: (i) They may contain inauthentic languages.

34

HUNG VUONG SUMMER C AMP-2010

Textbooks sometimes present inauthentic language since texts, dialogues and other aspects of content tend to be specially written to incorporate teaching points and are often not representative of real language use. (ii) They may distort content. Textbooks often present an idealized view of the world or fall to represent real issues. In order to make textbooks acceptable in many different contexts controversial topics are avoided and instead an idealized white middle-class view of the world is portrayed as the norm. (iii) They may not reflect students needs. Since textbooks are often written for global markets the y often do not reflect the interests and needs of students and hence may require adaptation. (vi) They can deskill teachers. If teachers use textbooks as the primary source of their teaching leaving the textbook and teachers manual to make the major instructional decisions for them the teacher role can become reduced to that of a technician whose primarily function is to present materials prepared by others.

In actual fact, the textbook, together with the syllabus, is the backbone that holds up a language course. In a textbook, teachers do find an appropriate sequence to follow and a support to base their daily teaching. Textbooks are good outlines that teachers can follow and adapt to their teaching, cultural, and institutional needs. Course books provide a source and a guide for students. If students have missed a class, have problems with a certain language aspect, want to review, practice or just know what comes ahead, they can find that information in the textbooks. According to ONeil (1990, p.151), textbooks are resource for staying in touch with the language. Students need a source to lean on, to consult and to revise anytime they feel suit. Moreover, in textbooks, students find information nicely presented and elaborated. Nobody can deny that the layout of most textbooks is unquestionably neat and with great graphic art. Students have in their textbooks wonderful pictures, charts, drawings, organization of information, and so on, presented in the best ways possible.

35

HUNG VUONG SUMMER C AMP-2010

To discuss the positive aspects of using a textbook, Ramirez Salas (2004, p.4) comments that a course without a textbook is very similar to a ship without a clear destination. It may nagativate days and nights and arrive at many ports, but if the route is not clearly set, important ports can never be reached, precious time can be wasted, and eventually, the ship can get completely lost. However, even good textbooks may have limitations when applied into different teaching situations. Therefore, when using a textbook, we should consider both the benefits and limitations, and if the textbook used in a program is judged to have some negative effects, remedial action should be taken, eg. by adapting it flexibly.

3.2. Why should we adapt textbooks? It is totally aware that most of our high school teachers are under considerable pressure because they are forced to comply with a syllabus and they teach many classes. They are also influenced by the attitude of the authority and those colleagues who see the textbook as resource that has to be closely followed. But the reality also forces teachers to find ways to spice up their classes without falling into a dangerous and tedious routine. Undoubtedly, one way to get students much more involved in their learning process and transforming them into active participants is by incorporating Materials Adaptation into our daily teaching. Ramirez Salas (as cited in Edge & Wharton, 1998, p.300) points out that experienced teachers do not tend to follow the script of a course book inflexibly. They add, delete and change the tasks at the planning stage, and they reshape their plans during the lesson in response to the interaction that takes place. Many times, a textbook presents the material in a way that does not fit the reality of the classroom or the current needs of the students. It is in this moment where the teacher has to define what to change, eliminate, add or extend. In fact, this is what keeps a class alive. If teachers over- use a textbook over a period of time, they will find themselves teaching the same type of activities in the same order repeatedly. In such a situation, even with good textbooks, students may find the study of English becoming a routine and thus less and less motivating. (Harmer,2003, p.257). Teachers should really avoid getting involved

36

HUNG VUONG SUMMER C AMP-2010

in this rigid sequence by providing students with supplementary material adapted to their needs. Another reason for teachers to adapt materials is the fact that even though textbook provide a framework, as cited by ONeil, this framework needs to be contextualized. Textbooks present materials, such as maps, flight and bus schedules, opening and closing times of public services, prices, names of cities, information about public or famous personalities that our students may not be familiar with.. Therefore, instead of having students look at fictitious facts about fictitious person, they could be given facts about a politician or entertainer, known to students. The fact that the students are talking about something as real as their home town makes the practice activity much more relevant, and engaging. Abstract knowledge constructs about common day-to-day experiences, the general framework of which, over time, can usefully be internalized by students. (Block, 1991, p.213-214) It can be denied that Materials Adaptation is very time-consuming. Sometimes teachers may even wonder what to prepare materials for if they are going to be used only once. But apart from this, teachers should realize that if it takes them long hours to create an activity, this activity should be recycled along the school year with different levels of difficulty and groups and it is worth trying when good results are obtained and motivation among students is increased, not to mention the teachers great feeling of satisfaction to present something created by himself. In this respect, Block (1991) comments that the personal touch in teacher-generated materials is highly appreciated by students. When students realize that the teacher has gone outside the course book and prepared something personally, they make remarks such Oh, you work hard. (p.214)

IV. Characteristics of Adapted Materials When talking about Materials Adaptation, there are some important features to take into a count in the process of adapting teaching materials. Among many characteristics mentioned by Tomlinson (1998), Ramirez Salas (2004, p.6) considers the four features, which are fundamental and worth discussing. The first one is the fact that materials should give an impact on the students and arouse learners curiosity, attention and interest. To achieve this, teachers should clearly know

37

HUNG VUONG SUMMER C AMP-2010

his/ her students and his/ her objectives to adapt an activity for a particular teaching aspect. These materials ought to be relevant, useful and focused on what students are learning at that point. A second characteristic is exposure to real language which is attained by giving students opportunities to use language in real- life communicative activities. By radio interviews, lectures, class surveys, spontaneous conversations, projects, interviews to other teachers, group discussions, students can be exposed to real language. The materials should also stimulate learners interaction with the input rather than just having passive reception of it. This does not necessarily mean that the learners should always produce language in response to the input, but it does mean that they should always do something mentally or physically in response to it. (Tomlinson, 1998, p.13) According to Ramirez Salas (2004, p.7) following directions, filling out an application form, chart or table with important information are examples or receptive activities in which students are processing authentic language. It is very important for teachers to ask themselves every time they adapt an activity how much real language students are using and/ or producing in that particular activity. If the answer is not much, then it is time to restructure or eliminate that activity. As a third feature, materials should address different learning styles and intelligences. If teachers know their students, they will design activities in which students can really feel at ease using their learning preferences and abilities. Of course, it would be difficult, though not impossible, to include in one activity all the styles and intelligences, but along the school year teachers can address one or two styles or intelligences in different activities. Students do not need to feel that language is not just a tight body of grammatical structures, rules and words, but a vehicle in which they can use their abilities and preferences to make their learning process much easier and enjoyable. The last but not the least, important feature of Materials Adaptation is the guidance towards students autonomy and independence. The latest trends in EFL teaching support and encourage the idea that students need to learn to be responsible for their own learning and to know that they can do activities in and out of class by themselves. Edge and Wharton (1998) pointed out that in order to achieve this in clas s teachers need to encourage the learners to reflect on what they are doing and why. (p.296). Giving students choices is a key element in making them autonomous and independent. Students

38

HUNG VUONG SUMMER C AMP-2010

need different alternatives to go over a specific task. Of course, t he one alternative students choose is likely to be closely related to their intelligences, learning styles and learning strategies. If teachers adapt teaching materials, they should offer choices, or at least, more than one possible way to complete the task.

V. Process of Materials Adaptation Jolly and Bolitho (1998, p.98-99) present a very self explanatory and easy-to- follow process to apply when adapting materials, which is as follow: 1. IDENTIFICATION: by the teacher or learner of a need to fulfill or a problem to solve by the creation of materials 2. EXPLORATION: of the area of need/ problem in terms of what language, what meaning, what functions, what skills? 3. CONTEXTUAL REALIZATION: of the proposed new material by the finding of suitable ideas, contexts or texts with which to work. 4. PEDAGOGICAL REALIZATION: of materials by the finding of appropriate exercises and activities and the writing of appropriate instructions for use. 5. PHYSICAL PRODUCTION: of materials, involving consideration of layout, type size, visuals, reproduction, tape length, ect. 6. STUDENTS USE OF MATERIALS 7. REWRITING OF MATERIALS BASED ON STUDENTS EVALUATION Ramirez Salas (2004, p.10) raises some questions necessary for adapting materials. Each of the questions corresponds to each step provided by Jolly and Bolitho: Step1: Do I really need other materials in this section? If the answer is YES (which in most of the cases happens) continue with the next question. Step 2. What can I do here? Step 3. Where can I find the information helpful to adapt something for this linguistic aspect? Step 4. What kind of exercises can I do here to exploit the idea or the text better? Step 5. What is the best way to present my activity? Step 6. Was the activity useful? Did I achieve my goal? Did students like it? Do I have to add, eliminate, correct, clarify, simplify, or rearrange something in this activity?

39

HUNG VUONG SUMMER C AMP-2010

Ramirez Salas (2004, p.7) considers that the process given by Jolly and Bolitho lacks one important phase, which is the revision and feedback from colleagues. This step would be placed just after the physical production of the materials. As language teachers, we need to share our ideas and receive feedback that can guide us to have a better performance. When creating or adapting something, we cannot see some flaws that may cause an unsuccessful outcome. Thats why, feedback, comments or ideas given by colleagues are important. They may radically change our perspective and consequently our entire activity. It is essential to have enough feedback from colleagues beforehand. Obviously, the process seems to be very simple at first glance. However, its application is difficult and time-consuming, but very rewarding. VI. Importance of Materials Layout In the process outlined in I.4, step 5 deals with the physical production of materials. This is a fundamental phase which has to be paid careful attention to. The materials layout must be a neatly presented product. But the word neatly does not refer just to the presentation as such, but to instructions, spelling, use of language, use of pictures, font size, and use of space. It is necessary to provide students with a layout which is full of mistakes with disorganized information and presents incorrect content. Instructions are important to the successful outcome of an activity. Instructions should be short, simple, concise and precise. They should never mislead learners or make them feel frustrated along the process of completing an activity. (Harmer, 2003, p.154) Another essential feature of an activity is spelling and language use. Students often copy or imitate everything given by teachers. If the model given is wrong, misspelled and with mistakes, what can teachers expect from students? Therefore, teachers should make sure that whatever given to students must be perfect. This is the moment in which colleagues are very helpful in pinpointing mistakes. Pictures are also important element in materials layout. They are commonly used in many activities. Pictures should be appealing, clear and big eno ugh. Also, pictures must be closely related to the topic of the activity. Indeed, pictures are not space fillers; they do not need to have a direct connection with the activity.

40

HUNG VUONG SUMMER C AMP-2010

If teachers are doing a lot of cut-and-paste from different sources, they should wipe out all black lines, unrelated information, or page numbers that may confuse students. Space and font size are also important in the adaptation of an activity. Students need space to write down and visual space between one step and the next one. In most of the cases, because of the budget limitations, teachers try to save money by putting a lot of information in small spaces, creating a very unattractive and overfilled page. However, an activity should be done with a comfortable reading font size (14 or 16) and with plenty of space for students to see the different sections, pictures, graphics, charts and tables of the activity. Students easily get lost when they are presented with too much information at once. They do need time to digest the information, and visual space provides this. It is a fact that sometimes teachers cannot type their activities and they have to present them in a handwritten way. A handwritten layout should be clear and neat to avoid confusion and time spent on endless clarifications.

CONCLUSION No one can deny that adapting materials is difficult and demanding. But it is necessary in order to offer students alternatives to practice the language in a more authentic and independent way. Materials Adaptation does not belong only to book writers. Teachers can create and adapt materials with a little extra time, motivation, creativity and love. It is a process that with practice and trial and error methods becomes more and more rewarding and necessary in our teaching contexts. It is also a way of keeping ourselves updated in new teaching trends and ideas. As Tomlinson states (1998) We need to make efforts to discover reliable and valid information about the sort of materials teachers and learners want to use. We need to innovate and e xperiment if we really want to find out how we could make language learning materials more effective and motivating. (p.158) In conclusion, textbooks play an important role in language teaching and provide a useful resource for both teachers and students. Good textbooks serve to turn the guidelines in the official government syllabus into a rich source of content, texts, and activities that would be beyond the capacities of most teachers to develop on their own. The use of textbooks should not be regarded as a resource that has to be closely followed, but as one of the

41

HUNG VUONG SUMMER C AMP-2010

many resources teachers can draw upon in creating effective lessons. It is the teachers responsibility to look for other ideas, to try them out and see what we can get and improve from them. That is the way we can do to lighten up our class and cast routine and boredom away. However, teachers need training and experience in adapting and modifying textbooks effectively and flexibly.

REFERENCES Block, D. (1991). Some Thoughts on DIY Materials Design. ELT Journal, 2 (45): 211-216 Edge, J; and Wharton, S. (1998). Autonomy and development: living in the materials world. In B. Tomlinson (Ed.) Materials Development in Language teaching. New York: Cambridge University Press. 295-310. Harmer, J. (2003). The practice of English language teaching. New York: Longman. Jolly, D. and Bolitho, R. (1998). A framework for materials writing. In B. Tomlinson (Ed.) Materials Development in Language teaching. New York: Cambridge University Press. 90-115. Kitao, K. (1998). Selecting and developing teaching/ learning materials. Retrieved on July 10th 2007, Kkitao@mail.doshisha.ac.jp Maley, A. (1998). Squaring the circle - reconciling materials as constrain with materials as empowerment. In B. Tomlinson ( Ed.) Materials Development in Language teaching. New York: Cambridge University Press. 279 -294. Marlene Ramirez Salas. English teachers as Materials Developers. Retrieved on July 10th 2007, marami@racsa.co.cr. ONeil, R. (1990). Why use textbooks? In Rossner & R. Bolitho (Eds). Currents of change in English Language Teaching. New York. Oxford University Press.148-156. Richards, C. The Role of Textbooks in a Language Program. Retrieved on July 10th 2007, http://www.teachingenglish.org.uk. Tomlinson, B. (1998). Materials Development in Language teaching. New York: Cambridge University Press.

42

HUNG VUONG SUMMER C AMP-2010

ENGLISH INTONATION FOR DIFFERENT TYPES OF QUESTIONS

Lao Cai Specializing Upper Secondary School I. Introduction Its widely believed that pronunciation is what makes up an accent. It may be that pronunciation is very important for an understandable accent. But it is intonation that gives the final touch that makes an accent native. It can be compared with a tune in music. It can go up or downwards. In English, the sentence intonation often indicates the mood of the speaker. Different tunes on the same word or phrase can send different messages about the speaker's feelings. Therefore, understanding English intonation patterns will increase not only your spoken English pronunciation competence, but your English listening comprehension as well. Instead of studying the intonation patterns for all kinds of sentences. In this special subject, I would like you to have a look at the intonation for questions only. I hope this topic will bring you some benefits in mastering your English speaking.

II.

Content

There are different intonation patterns used for different types of questions. In this topic I stay focus mainly on five popular types of English questions: Yes/No question; WhQuestion; Tag question; Choice or alternative question; and Declarative question.

1. Yes/No question A yes-no question, formally known as a polar question, is a question whose expected answer is either yes or no. Formally, they present an exclusive disjunction, a pair of alternatives of which only one is acceptable. Usually Yes/No questions have a rising intonation at the end of the sentence. This means that the speakers voice gets just a little higher as they finish the sentence. Example:

43

HUNG VUONG SUMMER C AMP-2010

Do you ever fall off?

Have you eaten yet?

Do you like beer? 1. 2. 3. 4. 5. 6. 7. 8. Below are some example sentences to practice. Are you ready to go? Could you please repeat that? Is it cold today? Am I interrupting? Are they cheaper in Japan? Are you a girl? Should we sell the fish? Is it time to go?

2. Wh-Question Wh-Questions or question-word questions, special questions are those that are formed with a question words such as what, which, when, where, why, who, whom, whose, how. They ask for specific information rather than an yes or no answer. Usually information questions (wh questions) have a rise/fall intonation at the end of the sentence. This means that the speaker's voice goes higher for a moment and then drops lower to end the sentence. Example:

When do you go riding?

44

HUNG VUONG SUMMER C AMP-2010

Who do you like in the fifth?

How did you spend your vacation?

However, in some cases wh-question can be said with non-fall intonation: a rise or less commonly, a fall-rise. This will make it more encouraging, gentle, kindly, sypathetic. Its named the encouraging rise. Example: How long have you been in How many people attended your 1. 2. 3. 4. 5. 6. 7. 8. 9. Newyork? party, Madam?

Below are some example sentences to practice. What time is it? Whats your name? Where do you live? How do you get to school? When should we go? Who ate the apple pie? Where can I find a bank? How old is Eric? How did they win?

3. Tag question In most languages, tag questions are more common in colloquial spoken usage than in formal written usage. They can be an indicator of politeness, emphasis, or irony. They may suggest confidence or lack of confidence; they may be confrontational or tentative, etc.

45

HUNG VUONG SUMMER C AMP-2010

We can change the meaning of a tag question with the musical pitch of our voice. There are two different kinds of tag questions. And both questions have different intonation patterns. With rising intonation, it sounds like a real question. But if our intonation falls, it sounds more like a statement that doesn't require a real answer. * Speaker is unsure of the answer. (speaker doesn't know whether Susan has eaten or not)

Example:

Susan ate, didn't she?

You're coming to the party,

aren't you?

* Speaker expects an agreement to the question. (speaker expects Susan has eaten)

Susan ate, didn't she?

This restaurant is terrible,

isn't it?

Besides, using rising or falling intonation can implicate other meanings. * Sometimes the rising tag also is used to create a confrontational effect: He was the best in the class, was he? (rising: the speaker is challenging this thesis, or perhaps expressing surprised interest) He was the best in the class, wasn't he? (falling: the speaker holds this opinion) Be careful, will you? (rising: expresses irritation) Take care, won't you? (falling: expresses concern)

46

HUNG VUONG SUMMER C AMP-2010

* Sometimes the same words may have different patterns depending on the situation or implication. You don't remember my name, do you? (rising: expresses surprise) You don't remember my name, do you? (falling: expresses amusement or resignation) Your name's Mary, isn't it? (rising: expresses uncertainty) Your name's Mary, isn't it? (falling: expresses confidence) * It is interesting that as an all-purpose tag the London set-phrase innit (for "isn't it") is only used with falling patterns: He doesn't know what he's doing, innit? He was the best in the class, innit? On the other hand, the adverbial tag questions (alright? OK? etc.) are almost always found with rising patterns. An occasional exception is surely.

4. Choice or alternative question

All choice questions have an 'or' in them and can either be open or closed question. Open and closed choice questions have different intonation patterns and require a different kind of answer.

An open question is a kind of yes or no question. An open choice question has

two possible intonation patterns.

Example:

A. Does she like chicken or meat?

B. Does she like chicken or meat?

47

HUNG VUONG SUMMER C AMP-2010

In the first, there is rising intonation after both choices. In the second, there is rising intonation only after the second choice. Both can be answered with a yes or no answer. Look at the questions below. The meaning for both is does she like one or the other choices or something else.

Possible ans wers:

Yes, she likes chicken. or Yes, she likes meat.

No, she doesn't like either. Yes, she likes both. No, but she likes fish.

A closed choice question has limited choices. It can not be answered with a yes or

no, but with one or the other choices or neither.

Example:

A. Does she like chicken or meat?

Possible ans wers : She likes both. Neither.

She likes meat. or She likes chicken.

5. Declarative question

Declarative questions are grammatically like statements. It is quite difficult for hearers to identify whether a question or a statement is intended. They can be identified as questions only by their intonation, or the pragmatics of the situations where they are used. They are usually said with a rise: a yes - no rise.

48

HUNG VUONG SUMMER C AMP-2010

Example:

You wil go to school by He took his

bike? (= Do you go to school by bike?)

identity card? (= Did he take his identity card?) did? (= I heard what you say => Its

I had an amazing experience. You surprising to me)

* Sometimes, they are said with a fall- rise. You didnt go and

tell him ? (= Does that mean that you told him?)

* And sometimes with falling intonation it shows that speaker is confused.

So we will be free by You mean he didnt turn III. Conclusion

six, then? (= Do you mean we will be free by six?) up?

All of these above information that I have just given surely can not cover the whole intonation used for questions. And you may find more implications for different intonations of these above questions. Hence, I would be very happy to welco me all of your feedbacks to make this topic completed.

49

HUNG VUONG SUMMER C AMP-2010

SRTATEGIES FOR ENGLISH LANGUAGE LEARNERS IN THE CLASSROOM

Duong Minh Khai Hung Vuong Specializing Upper Secondary School, Phu Tho It can be difficult to manage a class of students learning the English language, but there are many strategies that have proven effective in making this process trouble-free. This paper will review four of the most commonly used strategies and provide examples and the expected outcome of each strategy.

1.Pre-instruction Activities Pre-instruction activities can include graphic organizers, KWL (Know-Want-Learn) charts, semantic webbing, diagrams, and many other resources. This strategy provides an easy way to present new information to ELL students. Charts, graphs, and webs make it easier to show how information connects and can also be used to connect information that was previously learned. It also prepares the students to learn the new material that will be presented . Using the pre- instruction activity, students are introduced to new information. Students can also use the skills of making charts, graphs, or webs for their own projects. For example, the teacher may assign the students to write a short story about family. The student can use a graphic organizer to sort out their thoughts, ideas, and information that will be included in their story. The expected result of using pre- instruction activities is that students will feel less anxiety and will be better prepared for the new concepts that are introduced. It is also expected that the student will have something, as a result of the pre- instruction activity, to look back on throughout the lesson .

2. Visual Aides, Realia, Maps, Pictures, and M ultimedia Using concrete objects, pictures, and body movement will engage students, help them make connections and remember language that they are hearing and learning. Films,

50

HUNG VUONG SUMMER C AMP-2010

videos, and audio cassettes with books allow students to visualize and make connections to what is being taught. For example, watch a movie to help build prior knowledge about a topic you are teaching. This method lends itself to vocabulary and lessons on the parts of speech. This method would also be terrific for allowing students to hear dialog through the use of tapes, compact discs, and movies. It is important that students hear and see the language while learning it. Often, students will pick up on a concept easily if it is presented visually. This method provides examples to be looked at over and over, as well. This can result in a much smoother and productive learning experience.

3. Coope rative Groups and Peer Coaching Cooperative groups are valuable because they provide more frequent and extended opportunities to practice language than the traditional methods of teaching. The more advanced ELL students and assist those in need of more help and benefit from enacting of teacher. The student in need of help benefits from the support of a peer. Students tend to relate better to their peers and may learn more this way. Perhaps the greatest benefit of this method is that the teacher is free to circulate throughout the classroom and give help as needed. Peer coaching works in much the same way. Students receive individual attention from each other and benefit from interacting with one another. An example of a cooperative ELL classroom would be on in which students are divided into groups of four. Each group has students of high ability and students of lower ability. The students work within their cooperative groups on a daily basis. Peer coaching also works best if one student is of high ability and the other of lower ability. Cooperative group and peer learning are so effective because they motivate students to learn. Peers often listen to each other much more successfully that with the teacher . It also quells boredom within the classroom. The strategy of repeating and rephrasing information has been beneficial in many ELL classrooms. The teacher can take concepts that have already been mastered and place or rephrase them with new information. This provides a connection between old and new information. Teachers also need to use repetition to ensure that concepts will be

51

HUNG VUONG SUMMER C AMP-2010

remembered. To make sure that students are comprehending information, teachers need to repeat many times and rephrase before a student picks up on the concept. A great way to use the repeat and rephrase method would be during question and answer sessions. The teacher can ask a question of a student and if it is not understood can rephrase and repeat until the student masters the concept. For example, the teacher asks a student "What is the meaning of family?" The student may or may not understand the question in that form, but the teacher can then ask "What does family mean?" and the student may then be able to answer the question. The repeat and rephrase method lends itself to oral instruction . The obvious outcome of this method is that teachers will easily be able to check for understanding by rephrasing questions in different forms and expecting to receive the same answers. The repetition of concepts will make concepts stick in the mind .

4. Music and Jazz Chants One of the most successful ELL strategies is the use of music and jazz chants. They are so successful because music is universal to all languages. Songs are often easier to remember than just plain words and sentences. Jazz chants stimulate and appeal to multiple senses of learning. They also use the rhythmic presentation of the natural language which is important to successfully speaking English . Bilingual/ESL students need reading practice for fluency and pronunciation. Jazz chants and music are a fantastic way of practicing. This method is also excellent for memorization. For example, if a new ELL student is working to memorize the English alphabet, it will be much easier to do using the alphabet song we all learned in Kindergarten. This method can be used to introduce long concepts without anxiety. Students will remember the songs, more than likely, for the rest of their lives. Songs will help students learn long chunks of words. Students are active participants using this method and this makes the information easy to retain. Through the use of proven ELL strategies, learning English for non-native speakers is much less complicated. The techniques that have been reviewed above are essential to the success of an ELL classroom.

52

HUNG VUONG SUMMER C AMP-2010

PART II CLASSROOM PRACTICE RESOURCES

53

HUNG VUONG SUMMER C AMP-2010

TEST 1 PART I: VOCABULARY AND GRAMMAR. 1. Choose the correct ans wer by circling letter A, B , C or D 1. It is _____ that every pupil will have to follow the school timetable strictly. A. certainly B. sure C. obviously D. certain

2. Almost every household _____ a computer nowadays. A. have B. has C. has had D. had

3. Did you say that you _____ here three days ago? A. came B. had come C. have come D. come 4. Lets go ahead and do it now. Nothing ____ by waiting. A. accomplished accomplished 5. She studied hard and got a degree with ____ colors. A. scarlet B. red C. soaring D. flying B. accomplishes C. has accomplished D. will be

PART II: USE OF ENGLISH 1. Fill in each blank space in the following passage with one suitable word. School exams are, generally (1).., the first sign of test we take. They find out how much knowledge we have (2). But do they really show how intelligent we are? After all, isn't it a fact that some people who are very successful academically don't have any (3) sense? Intelligence is a speed (4).. which we can understand and (5). to new situations and it is usually tested by logic puzzles. Although scientists are now preparing (6).. computer technology that will be able to 'read our brains, for the time (7)., IQ tests are still the most popular ways of measuring intelligence. A person's IQ is their intelligence (8).. it is measured by this special test. The most common IQ tests are (9) by Mensa, an organization that was founded in England in 1946. By 1976 it (10).. 1,300 members in Britain. Today there are 44,000 in Britain and 100,000 worldwide (11).. in the US.

54

HUNG VUONG SUMMER C AMP-2010

People taking the test are judged in (12) ..to an average score of 100, and those who score over 148 are entitled to join Mensa. Anyone from the age of six can (13) the tests. All the questions are straightforward and most people can answer them if (14).. enough time. But that's the problem - the whole (15).. of the tests is that they're against the clock. 1. _________________ 4. _________________ 7. _________________ 10. ________________ 13. ________________ 2. ______________________3. ___________________ 5. ______________________6. ___________________ 8. ______________________9. ___________________ 11. _____________________12. __________________ 14. _____________________15. __________________

PART III: MAKING SENSE OF WHAT THE SPEAKER SAYS. 1. Choose the best answe r A, B , C or D to each question. 1. Man: I missed the weather on the radio this morning. Woman: The forecast wasnt certain, but I decided to carry an umbrella with me anyway. What does the woman imply? A. The forecast said it might rain morning C. Weather is reported in the evening D. The umbrella belongs to the man. B. No forecast was broadcast that

2. Girl: I hate to bother you, but your television is really loud. Boy: Sorry, I didnt realize that you could hear it next door.

What will the boy will do? A. Turn down the volume of the television television C. Begin looking for a new apartment future. D. Watch more television in the B. Change the channel on the

3. Man: Look at the cost of these long-distance calls. Woman: We really cant afford that much every month/ What are the speakers talking about?

55

HUNG VUONG SUMMER C AMP-2010

A. Tuition for school C. A telephone bill

B. A sale on clothing D. Vacation plans

4. Father: The light in the living room doesnt work Mother: The bulb is probably burned out. Ill buy one tomorrow. What does the mother imply? A. She doesnt like shopping C. She doesnt know how to change bulb B. She doesnt have a light bulb D. She prefers to eat in the dark.

5. Girl: We plan to go to a jazz club this evening. Would you like to come with us? Boy: No thank. I wouldnt be caught dead in a jazz club. What does the boy mean? A. Hes not feeling well C. He has other plans B. He doesnt want to go D. Hes been to many clubs.

PART IV: READING COMPREHENSION 1. Read the following passages then ans wer the questions and choose the correct ans wers.

A. Although speech is the most advanced form of communication, there are many ways of communicating without using speech. Signals, signs symbols and gestures may be found in every known culture . The basic function of a signal is to impinge upon the environment in such a way that it attracts attention, for example, the dots and the dashes of a telegraph circuit. Coded to refer to speech, the potential for communication is very great. Less adaptable to the codification of words, signs also contain meaning in and of themselves. A stop sign or a barber pole conveys meaning quickly and conveniently. Symbols are more difficult to describe than either signals or signs because of their intricate relations hip with the receivers cultural perceptions. In some cultures, applauding in a theater provides performers with an auditory symbol of approval. Gestures such as waving and handshaking also communicate certain cultural massages.

56

HUNG VUONG SUMMER C AMP-2010

Although signals, signs, symbols and gestures are very useful , they do have a major disadvantage. They usually do not allow ideas to be shared without the sender being directly adjacent to the receiver. As a result means of communication intended to be used for long distances and extended period are based upon speech Radio, television and the telephone are only a few . 1.Which of the following would be the best title for this passage? A. Signs and Signal B. Gesture C. Communication D. Speech

2. What does the author say about speech? 3. According to the passage, what is a signal? A. The most difficult form of communication to describe B. A form of communication which may be used across long distances. C. A form of communication that interrupts the environment . D. The form of communication most related to cultural perceptions. 4. The phrase impinge upon is closed in meaning to_______ A. intrude B. improve C. vary D. prohibit 5 . The word it in bold refers to ________ A. function B. signal C. environment D. way 6. The word potential could be best replaced by _______. A. range B. advantage C. organization D. possibility 7. The word intricate could be best replaced by which of the following? A. inefficient B. complicated C. historical D. uncertain

8. Applauding was cited as an example of ______. A. a signal B. sign C. a symbol D. a gesture

9. Why were the telephone , radio and TV invented ? A. people were unable to understand signs, symbols and signals . B. People wanted to communicate across long distances . C. People believed that signs , signals and symbols were obsolete . D. People wanted new form of entertainment . 10. It may be concluded from the passage that __________. A. signs , signals ,symbols and gestures are forms of communication.

57

HUNG VUONG SUMMER C AMP-2010

B. symbols are very easy to define and interpret . C. Only some cultures have signals , signs and symbols. D. Waving and handshaking are not related to culture.

B. In the 1960s, The Beatles were probably the most famous pop group in the whole world. Since then, there have been a great many groups that have achieved enormous fame, so it is perhaps difficult now to imagine how sensational The Beatles were at that time. They were four boys from the north of England and none of them had any training in music. They started by performing and recording songs by black Americans and they had some success with these songs. Then they started writing their own songs and that was when they became really popular. The Beatles changed pop music. They were the first pop group to achieve great success from songs they had written themselves. After that it became common for groups and singers to write their own songs. The Beatles did not have a long career. Their first hit record was in 1963 and they split up in 1970. They stopped doing live performances in 1966 because it had become too dangerous for them their fans were so excited that they surrounded them and tried to take their clothes as souvenirs! However, today some of their songs remain as famous as they were when they first came out. Throughout the world many people can sing part of a Beatles song if you ask them. 1. The passage is mainly about A. How the Beatles became more successful than other groups C. The Beatles fame and success B. Why the Beatles split up after 7 years D. Many peoples ability to sing a Beatles song 2. The four boys of the Beatles A. Came from the same family B. Were at the same age C. Came from a town in the north of England D. Received good training in music

3. The word sensational is closest in meaning to A. Notorious B. Bad C. Shocking D. Popular

4. The first songs of the Beatles were

58

HUNG VUONG SUMMER C AMP-2010

A. Written by themselves B. Broadcast on the radio 5. What is not true about the Beatles? A. The members had no training in music B. They had a long stable career

C. Paid a lot of money D. Written by black Americans

C. They became famous when they wrote their own songs D. They were afraid of being hurt by fans 6. The Beatles stopped their live performances because A. They had earned enough money C. They spent more time writing their own songs B. They did not want to work with each other D. They were afraid of being hurt by fans 7. The year 1970 was the time when A. They split up B. They changed pop music 8. What the fans of the Beatles often did was A. Sing together with them B. Take their clothes as souvenirs 9. Some songs of the Beatles now A. Are still famous as they used to be B. Became too old to sing 10. The tone of the passage is that of C. Are sung by crazy fans D. Are the most famous C. Ask them to write more songs D. Ask them why they should separate C. They started their career D. They stopped singing live

C. America online is one of the big names on the Internet, and unlike many other digital companies, it actually makes a profit. But the company which its rivals call the Cybercockroach was launched only in 1992. Before that it was a small firm called Control Video games. Then Steve Case, a former Pizza Hut marketing executive arrived and took the company online. Innovative, fast moving, and user- friendly, America Online appeals to people who want to surf the Internet, but who more do not have a lot of experience. For

59

HUNG VUONG SUMMER C AMP-2010

the same reason techies, people who think they are more expert with computers, look down on America Online and its users. Recently American Online (or AOL, as it calls itself) joined with Time Warner a multi- million dollar movie and magazine company to create a multimedia giant. Now, AOL has begun to expand abroad. In many European countries, including the United Kingdom, it is hard to buy a company magazine that does not have a free AOL introductory offer. The company also puts advertisements onto the television, and employs people to hand out its free introductory disks at places like train stations. As the internet gets faster AOL is changing. With many homes getting high-speed connections through he fiber optic cables or the new ADSL technology, the Cyber-cockroach will have to show that, like real cockroaches, it can survive in almost any environment. 1. What is the passage about? .. 2. Who does Steve Case work for? A. AOL B. Pizza Hut C. Control Video Corporation D. None of these 3. How do Techies feel about America Online? A. They think it is a Cyber-Cockroach. B. They think it is for experts. D. They feel superior to its users.

C. They think it is a movie and magazine company. 4. People who use America Online are probably _____. A. Video game players C. Movie fans B. Techies

D. People new to the internet

5. America Online is an unusual digital company because _____. A. It used to make video games C. It makes money B. It is innovative D. It has joined with another company

6. Which marketing idea is not mentioned? A. Advertisements on the internet. C. Free disks in journals B. Advertisements on TV. D. People giving disks away.

7. What does the article say about AOLs future? A. It will do well. C. It will face challenges. 8. This passage is about _____. B. It will do badly. D. The article doesnt say.

60

HUNG VUONG SUMMER C AMP-2010

A. Technology C. Computer users 9. Cyber-Cockroach is _____. A. America Online B. Its rival

B. A history of the Internet D. A successful business

C. the internet

D. All is wrong

10. The word optic means _____. A. light B. eye C. communication D. thread

PART V: WRITING. I. Complete the second sentence in such a way that it is almost the same as the first. 1. Im sorry that I didnt finish my homework last night > I wish................................................................................................................................... 2. Keeping calm is one of the secrets of passing your driving test. > As long as ........................................................................................................................... 3. He often spends 15 minutes leading the buffalo to the field. > It often takes ....................................................................................................................... 4. Who feeds the tigers at the zoo every day ? > By ........................................................................................................................................ 5. Henry regretted buying second hand-car . > Henry wishes ....................................................................................................................... 6. People who were there didnt notice anything unusual. > No one who.......................................................................................................................... 7. This is the most delicious cake Ive ever tasted. > I have never.......................................................................................................................... 8. She left university two years ago . > It is ....................................................................................................................................... 9. Newspapers are sent to China by train . (Change it into active form) > ............................................................................................................................................. 10. The mechanic serviced my car yesterday . > I had...................................................................................................................................... II. Write a paragraph of 150-200 words discussing the uses of Internet in our daily life

61

HUNG VUONG SUMMER C AMP-2010

TEST 2 PART I: PHONETICS 1. Choose the word whose underlined part is pronounced differently from the rest in each group. 1. A. curent 2. A. further 3. A. mountain 4. A. tiny 5. A. coast B. turn B. fertile B. southern B. island B. lost C. fun C. heard C. mouth C. river C. most D. under D. mother D. count D. wind D. whole

2. Pick out the word whose stress is placed differently 6. A. disappear 7. A. consume 8. A. curtain 9. A. distance 10. A. tremendous B. engineer B. reflect B. enclose B. glorious B. enormous C. education C. local C. contain C. exciting C. serious D. attachment D. between D. remember D. journey D. describe

PART II: VOCABULARY AND GRAMMAR 1. Circle the letter of the best answe r. 1. Firemen rescued several people from the floor or the blazing building. A. high B. top C. basement D. low 2. When the electricity failed, he a match to find the candles. A. rubbed B. scratched C. struck D. started 3. She put speaking to him as long as possible A. off B. over C. away D. back 4. She wondered her father looked like now, after so many years away. A. how B. whose C. that D. what 5. She her neighbors children for the broken window. A. accused B. complained C. blamed D. denied 6. He filled in the necessary forms and for the job.

62

HUNG VUONG SUMMER C AMP-2010

A. appealed

B. asked

C. requested

D. applied

7. He says hes been to restaurant in town. A. many B. every C. all D. most 8. She hasnt written to me A. already B. yet C. never D. any longer 9. She is traveling to work by train today because her car is being A. stopped B. broken C. serviced D. rented 10. He tried every key in turn but not fitted A. each B. one C. any D. none 11. Dont you get tired watching TV every night? A. with A. didnt get B. by B. didnt put C. of C. didnt have D. at D. didnt take 12. I asked Grill what time it was but she said she a watch. 13. The film lasted three hours with of 15 minutes between part one and part two A. an interval B. a pause C. a stop D. an interruption 14. If you saw a lawyer, hed you to take legal action. A. suggest B. warm C. consider D. advise 15. Im very busy at the moment so it may take a .. time to answer you letters. A. little B. few C. small D. come 16. Ive known him .. I left college A. when B. during C. until D. since 17. He was wearing a heavy overcoat to .. himself against the cold. A. Conceal B. protect C. cover D. wrap 18. It took Michael a long time to find a pair of shoes that .. him. A. liked B. fitted C. agreed D. matched 19. ..the traffic was bad, I arrived on time. A. Although B. In spite of C. Despite D. Even 20. The discovery was a major .. for research workers. A. breakthrough B. breakdown C. break-in D. breakout

63

HUNG VUONG SUMMER C AMP-2010

2. Use the proper tense/form of verbs in pare ntheses. Write your ans wer in the space provided. 1.. 2. 3. 4. 5. 6 7 8.. 9 10..

1 He looked frightened as if he (1. see) _____ a ghost 2 A moment please. I (2. be) _____ not ready yet but I (3. be) _____ in a couple of minutes. 3 The car broke down. But for that we _______ (4. arrive) in time. 4 I dont mind her_____(5. appoint) above me. 5 An eyewitness described how ten people ______ (6. kill) in the fire. 6 The last time Bill________(7.see) in the public, he________(8.wear) a red suit. 7 I (9. need, cook) ______________dinner. Just as it was ready, Chris and June _______ (10. phone) to say that they couldnt come to eat

3. Error correction: Circle the letter which is incorrect in the following sentences. 1. The number of time he spends gardening is increasing every day. A B C D

2. It would be both noticed and appreciating if you could finish A the work before you leave. C D B

3. The suits were hanged in the closet when they were returned from the cleaners. A B C D

4. A vast quantity of radioactive material is made when a hydrogen bomb explode. A B C D

5. Commercial letters of credit are often used to financing export trade, but they can A B C

64

HUNG VUONG SUMMER C AMP-2010

have other uses. D

4. Complete the letter by writing one word in each gap. Exam Tips When the day comes give yourself plenty of time to do everything: have breakfast but don't drink (1) ... much; go to the toilet; arrive on time, but not too early or you will find yourself getting more and more nervous while you wait to start. Try no t to talk (2) ... the exam before you go in. In the exam, calm (3) ... down by breathing deeply and thinking positively. Read (4) ... exam questions carefully and underline all of the key instruction words (5) ... indicate how the questions should (6) ... answered. If possible start with the ones (7) ... can do easily to give you confidence. Remember what you've learnt from practising questions and doing mock exams previously and plan your use of time. Don't panic (8) ... everyone around you seems to start writing furiously straight away and don't be tempted to follow their example. Finally, after the exam, don't join in a discussion about (9) ... everyone else did, (10) ... you want to frighten yourself, and drain your self-confidence for the next exam. Above (11) ..., remember that exams are not designed to catch you out, (12) ... to find out what you know, what you understand and what you can do. Your answers: 1. 6. 11. 2. 7. 12. 3. 8. 4. 9. 5. 10.

5. Give the right form of the word in the capital letters 1. In his .......................... to make himself understood, he spoke very slowly. ANXIOUS 2. John Irving is my favorite ________________ and Ive just finished his latest book. NOVEL 3. Our holiday was quite ________________ because we took a tent instead of staying in a hotel and we hired bikes instead of a car. EXPENSE

65

HUNG VUONG SUMMER C AMP-2010

4 Sarah has a degree in law and a diploma, so her ________________ for the job are very good. QUALIFY 5 Beckhams knee ________________ prevented him from playing in the Cup Final last year. INJURED

Your answers: 1. 2. 3. 4. 5.

6. Match the questions/statement (1-10) with the responses (a-j) below. a. Its my pleasure. b. Has she really? How wonderful! c. No, of course not.

1. Excuse me, is anyone sitting here? 2. I didnt know Tam gave up that job. 3. Im sorry Im late. The traffic was terrible. 4. Thanks for the present, my darling. 5. My grandfathers much better now. 6. Have you got a light, please? 7. Do you mind if I turn the light on? 8. I was wondering if you could give me a hand. 9. My daughters just had her first book accepted. 10. Pass me the sugar please!

d. Here you are. e. Id rather you didnt, my sweetheart! f. I didnt either g. Oh, thats all right. h. Yes, of course. i. Im sorry, I dont smoke. j. Oh, Im so pleased to hear it

Your answers: 1. 6. 2. 7. 3. 8. 4. 9. 5. 10.

PART III: READING COMPREHENSION

66

HUNG VUONG SUMMER C AMP-2010

1. Read the text below and decide which ans wer A, B, C or D best fits each space.

Family History In an age when technology is developing faster than ever before, many people are being.(1) to idea of looking back into the past. One way they can do this is by investigating their own family history. They can try to..(2) out more about where their families came from and what they did. This is now a fast- growing hobby, especially in countries with a (3) short story, like Australia and the United States. It is...(4)thing to spend some time..(5) through a book on family history and to make the (6).to investigate your own familys past. It is (7)another to carry out the research work successfully. It is easy to set about it in a disorganized way and (8).yourself many problems which could have been (9)..with a little forward planning. If your own family stories tell you that you are..(10) with a famous character, whether hero or criminal, do not let this idea take over your research. Just (11).. it as an interesting possibility. A simple system (12).. collecting and storing your information will be adequate to start with, a more complex one may only get in your(13). The most important thing, though, is to (14).started. Who knows what you..(15) find?

1. a. pushed 2.a. lay 3.a. fairly 4.a. a 5.a. seeing 6.a. idea 7.a. quite 8.a. produce 9.a. missed 10.a. connected 11.a. treat

b. attracted b. make b. greatly b. one b. moving b. plan b. just b. cause b. lost b. joined b. control

c. fetched c. put c. mostly c. no c. going c. purpose c. more c. build c. avoided c. attached c. contact

d. brought d. find d. widely d. some d. living d. decision d. even d. create d. escaped d. related d. direct

67

HUNG VUONG SUMMER C AMP-2010

12.a. with 13.a. track 14.a. get 15.a. should

b. by b. path b. appear b. might

c. for c. road c. be c. ought

d. through d. way d. feel d. must

2. Read the passage carefully and then choose the best answer to each sentence. Crows are probably the most frequently met and easily identifiable members of the native fauna of the United State. The great number of tales, legends, and myths about these bird indicates that people have been exceptionally interested in them for a long time. On the other hand, when it comes to substantive-particularly behavioral- information, crows are less well known than many comparably common species and for that matter, not a few quite uncommon ones: The endangered California condor, to cite one obvious example. There are practical reasons for this. Crows are notoriously poor and aggravating subjects for field research. Keen observers and quick learners, they are astute about the intentions of other creatures, including researchers, and adept at avoiding them. Because they are no numerous, active, and monochromatic, it is difficult to distinguish one crow from another. Bands, radio transmitters, or other identifying devices can be attached to them, but this of course requires catching live crowns, who are among the wariest and most untrappable of birds. Technical difficulties aside, crow research is daunting because the wa ys of these birds are so complex and various. As preeminent generalists, members of these birds are so complex and various. As preeminent generalists, members of this species ingeniously exploit a great range of habitats and resources, and they can quickly adjust to changes in their circumstances. Being so educable, individual birds have markedly different interests and inclinations, strategies and scams. For example, one pet crow learned how to let a dog out of its kennel by pulling the pin on the door. When the dog escaped, the bird went into the kennel and ate its food. 1. What is the main topic of the passage? A. The ways in which crows differ from other common birds. B. The myths and legends about crows. C. The characteristics that make crows difficult to study.

68

HUNG VUONG SUMMER C AMP-2010

D. The existing methods for investigating crow behavior. 2. According to the first paragraph, what evidence is there that crows have intersted people for a long time? A. The large number of stories about crows. B. The frequency with which crows are sighted. C. The amount of research that has been conducted on crows. D. The ease with which crows are identified. 3. In line 6, the author mentions the endangered California condor as an example of a species that is .................... . A. small than the crow. B. easily identifiable. C. featured in legends. D. very rare. 4. According to the second paragraph, crows are poor subjects for field research for all of the following reasins EXCEPT A. They can successfully avoid observers. B. They are hard to distinguish from one another. C. They can be quite aggressive. D. They are difficult to catch. 5. In the second paragraph, the author implies that using radio transmitters would allow a researcher who studies crows to ...................... . A. identify individual crows. B. follow flocks of crows over long distances. C. record the times when crows are most active. D. help crows that become sick or injured.

Your answers: 1.. 2. 3 4 5

3. Choose from the list the sentence which best summarizes each part of the article. There is an extra sentence which you do not need.

69

HUNG VUONG SUMMER C AMP-2010

A. B. C. D.

Research is being done into electric cars. People refuse to give up their cars in cities One answer is to persuade people to buy electric cars. Cities where people depend heavily on cars have the worst

problems. E. F. Air pollution appears to be cause of illness. Air pollution is now a worry for everyone.

1. Calculating the number of people who become unwell or even die as a result of air pollution is very difficult. But recent studies of the effects of car fumes suggest that the health risks may have their roots from air pollution. 2. Suddenly, urban air pollution is no longer a subject just for environmentalists but a cause of widespread public concern. Ordinary people are beginning to sit up and take notice. 3. There are numerous proposed ways of dealing with the problem: one of the most radical is to slowly stop using the internal combustion engine and to use instead the electronic motor powered from a large battery pack. A lot of money is now being invested and many studies have been carried out by car and battery manufacturers to create clean vehicles. 4. Much of the pressure has come from the land where the car is king- California. The United States has no public transport to speak of ( the major car- markers actively contributed to its destruction) so the car is the average Americans only practical mean of daily transport. But some US cities, Los Angeles in particular, are paying a high price for this overreliance. LAs famous smog- trapped by the natural bowl of the nearby mountains, are the result of reactions between the chemicals which come from the citys millions of car exhausts.

70

HUNG VUONG SUMMER C AMP-2010

5. Because of this serious pollution problem. California has for over 20 years set tough pollution laws. Even so, LAs smog problem has not been solved. So now, the sunshine state has taken the first step towards removing the internal compulsion engine altogether from its roads. From 1998, all cars- makers who sell their cars in California will have to offer a proportion of electric cars for sale. In 1998 the proportion of electric cars offered must be two percent, rising to five percent by 2003 and to ten percent by 2005.

Your answers: 1.. 2..3.. 4.. 5..

PART IV. WRITING 1. For each of the sentences below, write a new sentence as similar as possible in meaning to the original sentence, but using the word given. This word must not be altered in any way. 1. The teachers agreed to introduce the new methods. (agreement) Dickens last novel was unfinished when he died. (without) 2. 3. All the hostages were released yesterday by the kidnappers. (let) 4. John was shocked to hear that he had failed his driving test. (came) 5. That sort of behavior is deplorable, in my opinion. (approve)

2. Rewrite the following sentences so that the meaning stays the same. 1. The government has been reviewing their immigration policy for some time. The governments................................................................................................................ 2. A new flu vaccine has been on trial since the beginning of the year. They.....................................................................................................................................

71

HUNG VUONG SUMMER C AMP-2010

3.

If anyone succeeds in solving the problem, it would be him.

He is the most...................................................................................................................... 4. He met Jane, whom he later married, when he was at Cambridge. He met Jane, who was......................................................................................................... 5. His behavior is beginning to annoy me more and more. I am beginning to.................................................................................................................

3. The re is a fact that a number of parents now increasingly desire to send their children to study overseas. What are the reasons for this trend? You should write a paragraph of about 150 words on this topic.

72

HUNG VUONG SUMMER C AMP-2010

TEST 3

Part I: VOCABULARY AND GRAMMAR 1. Choose the best word or phrase to complete each sentence (1-25). Write A, B, C, or D in the space provided according to your choice. 1. Prices have been .. A. sized up B. set down by increases in production. C. tied up D. brought down

2. She did not study hard, so she had to take the . A. consequences B. sequences C. effects D. outcome 3. The book had a great .. on its readers. A. affection 4. John . A. examined 5. She . A. strayed B. stroke C. contentment D. impact

the results of the election with amazing accuracy. B. overlooked C. predicted D. entailed

from scolding the children until the guests left. B. refrained C. stopped D. prevented

6. Did you mean me to keep the receipt ? Im afraid I have A. stored it up B. taken it apart C. torn it up D. cut it out 7. I have just been in to see if I can be . any assistance to them. A. of B. with C. on D. at 8. .for an explanation of the social customs of this country, I would definitely find it difficult. A. If asked B. If asking C. Asking D. Asked 9. This is a subject .. we might argue for a long time. A. which B. about which C. what D. about what 10. These shoes are comfortable .. pretty. A. except for B. more than C. rather than D. as to 11. As soon as the workmen left, she began to clear up the .. A. mess B. trouble C. confusion D. destruction 12. I wonder who will as secretary of our club next year. A. take up B. take over C. take on D. take to

73

HUNG VUONG SUMMER C AMP-2010

13. He had a cold and couldnt go to the party, so I bought him a cake to make up for his . A. decline B. demand C. digestion D. disappointment 14. The seating .of our auditorium is 1,000 A. capacity B. condition C. degree D. accommodation 15. The firemen acted .. and prevented the fire from spreading. A. promptly B. partially C. voluptuously D. elegantly

16. No sooner had we left the house................ it started snowing. A.and B.than C. when D. that 17. During World War II, tea was .and we had to drink plain water instead. A. few B. abundant C. nutritious D. scarce

18. Several cars, ................owners had parked them under the trees, were damaged. A.their B. of which C. whom D. whose

19. It was very good ................you to give up so much of your time. A.of B.for C. with D. to

20. The play was not................ what we had expected. A.just B.absolutely C.at all D. very 21. Youll have to pay fifty dollars in cash. they give the machine to you. A. as B. until C. before D. while

22. The minister ................to say whether all the coal mines would be closed. A. refused B. avoided C. denied D. bothered 23. He ..come at 10 oclock, but now he comes at noon. A. was used to B. used C. was used D. used to

24. At the end of the match the players were ................exhausted. A. solely B. utterly C. actually D. merely

25................ you should do first is make a list of all the things you have to do. A. That B. What C. As D. If

Write your answers in the following space. 1. 6. 2. 7. 3. 8. 4. 9. 5. 10.

74

HUNG VUONG SUMMER C AMP-2010

11. 16 21

12. 17 22

13. 18 23

14. 19 24

15. 20 25

Part II. USE OF ENGLISH 1. Read through this articles about volcanoes and think of a word which best fits

each space. Use only one word in each space. An Unchanging Planet ? If you think of Earth (26)............. a stable and unchanging planet, think again. Nearly five billions years (27) ............. it was first formed, the Earth is still developing (28)............. alarming ways. Unlike earthquakes that strike (29) ..........warning, volcanoes build up (30) ............ months and are usually easier to predict. But their spectacular climax is no (31) .............. devastating. A volcano can erupt in many different ways (32) ................ it can spill out a variety of materials. Mild eruptions spurt gas, steam and hot water and are (33).............. geysers. Larger volcanoes shoot out ash and large chunks of hot rock into (34)............ atmosphere, and enormous fountains of glowing red hot lava that flow (35) ............ the sides the volcanoes. This liquid lava quickly thickens into a steaming s ticky carpet (36) ............... can travel 150 km before it stops and turns solid. Lava floods (37) .................fire to and destroy (38)................ that stands in their way. Famously, in AD 79, the Roman city of Pompeii (39) ............. covered in lava and ash, preserving buildings and some of their contents to (40) ............ day.

Write your answers in the following space. 26. 31. 36. 27. 32. 37 28. 33. 38 29. 34. 39. 30. 35. 40.

2. Read the text below. Use the word given in capitals at the end of each line to form a word that fits in the space in the same line. The re is an example at the beginning (0) Pablo Picasso was born in Spain. As a child, he was very (0) talented. At the age of 19, he went to Paris and experimented with (41) ............... TALENT DIFFER

75

HUNG VUONG SUMMER C AMP-2010

styles, from sad paintings in blue to more (42).............. ones of circuses in reds and pinks. Picasso very quickly became (43)...............He was constantly looking for new (44)............... He became interested in (45)............ masks, which were being shown in Europe for the first time. He particularly liked the simple but (46)....... way they distorted the human face. He began to use (47)............. shapes to build up an image this was the (48)............ of cubism. Even though his cubist pictures are (49)............., we still understand what they are supposed to be (50)...............

CHEER SUCCEED INSPIRE AFRICA EXPRESSION ANGLE BEGIN REALISM SYMBOL

Write your answers in the following space. 41. 46. 42. 47. 43. 48. 44. 49. 45. 50.

Part III. MAKING SENSE OF WHAT THE SPEAKER SAYS 1. Read each of the following short conversation and then try to unde rstand what the speaker means to say by choosing one answe r A, B, C, or D to the question in italic afte r the conversation. Then write your ans wer in the space provided. 51. Minh. I feel a draft. Is the window shut? Nam. When they left, they did not close the door. What does Nam mean? A. B. C. D. They gave me a lift He did not stand close to the door. He walked through the door on the left. The door was left open

52.John. Rich surely spends a lot of time studying. Mary. Had she known that the math class requires so much work, she would have taken it next year. What does Mary mean? A. Rich students dont take math.

76

HUNG VUONG SUMMER C AMP-2010

B. C. D.

Rich didnt know much math. Rich is taking a math course Rich should take math next year.

53. Peter. What were those people asking you? Daisy. They were wondering if the picture is for sale. What does Daisy mean? A. B. C. D. They wanted to know if they could buy the picture. They were wondering around looking for a picture. They stopped by the picture gallery to ask questions They thought that the picture was wonderful.

54. Hung. Slow down. You are passing every car on the road. Mai. Most drivers dont observe the speed limit unless they think the police will stop them. What does Mai mean? A. B. C. D. The police stop most drivers. The speed limit is unreasonable. Drivers do not watch the traffic carefully. Few people drive within the speed limit.

55. Ba. These students dont even know what the grading policy is. Huong. When the teacher explained the course requirements, they were probably day dreaming. What does Huong mean? A. B. C. D. The teacher taught about dreams. The students are required to take the course. The lectures on dreams was a great success. Hardly anyone was listening to the teacher.

Write your answers in the following space.

77

HUNG VUONG SUMMER C AMP-2010

51.

52.

53.

54.

55.

PART IV. SENTENCE WRITING 1. Rewrite each sentence so that it contains the word in capitals, and so that the meaning stays the same. 56. I cant find the answer without a calculator. ................................................................................................................... 57. His smooth manner didnt deceive us. ................................................................................................................... 58. I am sure City will win. ................................................................................................................... 59. Everyone who spoke to the victim is a suspect. ................................................................................................................... 60. We get on very well with our next-door neighbors. ................................................................................................................... 61. I should really be going now. ................................................................................................................... 62. Foolishly, I paid all the money before collecting the goods. ................................................................................................................... 63. Robert had no idea of his next move. ................................................................................................................... 64. It was only when I checked that I noticed the type was flat. ................................................................................................................... 65. Please inform the relevant authorities at one. ................................................................................................................... DELAY DID DO WHICH TIME TERMS UNDER BOUND TAKEN OUT

PART V. READING COMPREHENSION 1. Read and ans wer the questions by choosing one best answe r, A, B, C, or D. Equality of opportunity in education and training, employment and promotion, business, and entrepreneurship is an ideal that most people would accept. A person who

78

HUNG VUONG SUMMER C AMP-2010

hasnt been given the opportunity cannot contribute economically or socially to the full extent that she or he is able. Such a perso n cannot enjoy the social or economic benefits that she or he is capable of earning. This lost opportunity for a person is an economic and moral loss to society. Today, a large group of people is denied equality of opportunity and this group, in most societies, is women. In the past twenty to thirty years, much has been done to reduce inequalities in employment and pay in Western countries. Nevertheless, much remains to be done in these Western nations. There are still few women in high- level positions in employment, fewer women than men enter universities and other post-secondary institutions, and women are mostly concentrated, still, in five traditional jobs, nursing and other health care-related paraprofessional occupations, teaching, secretarial/c lerical, retail trade and general service jobs.

Indicate whether the following statements are true or false. 59. 60. 61. Most people think that everyone should have the same chances. Unequal opportunity means a person cannot do what he or she is capable of doing. Conditions for working women in Western countries have not changed much in the

past twenty to thirty years. 62. 63. Only in Third World countries are there still pay and employment inequalities. Jobs in dental hygiene, nursing and social work are predominantly filled by

women.

Cirle the letter next to the best ans wer. 64. Equal chances at education and training give people: A) the possibility of having a good life. B) the possibility of helping society C) A+B 65. When everyone doesnt reach her or his potential, there is: A) an economic loss for society B) an immoral action C) A+B

79

HUNG VUONG SUMMER C AMP-2010

66.

Women get fewer opportunities than men in: A) some societies B) all societies C) many societies

67.

The number of women who enter universities is: A) greater than those in high- level positions B) less than those in high- level positions C) the same as those in high- level positions

68.

Women workers form the majority: A) offices, clinics and cafeterias B) banks, law firm and laundries

C) hospitals, clinics and stock exchanges

Write your answers in the following space. 66. 71. 67 72. 68. 73. 69. 74. 70 75.

2. Proble m solving (76-80) Allibhai Sayani is a businessman living in the Third World with his wife and their four teenage children- two sons and two daughters. He would like to send all four children to the west to get a university education. Unfortunately, he only has enough money to send two and decided to send his two daughters. You conduct a conversation in which you discuss whether Allibhais decision is a good one and give your reasons. Write your answers in the following space

3. Essay Writing Write a paragraph on the following topic Women were better off in their role in a traditional family than they are today. Do you agree?

80

HUNG VUONG SUMMER C AMP-2010

TEST 4

PART I. VOCABULARY AND GRAMMAR

1. Circle the best option to complete the sentence. 1. A. 2. A. 3. She caught a ---- of the thief as he made his get-away. glance B. sight C. glimpse D. flash

The little boy was continually ---- the ornaments. tripping up B. falling down C. breaking up D. knocking over All my children have grown up and ---- the nest. They havent returned home to

visit me for ages. A. 4. flown B. thrown C. escaped D. eaten

Why are they having those meetings behind closed doors? I am afraid there is

something ---- going on! A. 5. A. 6. A. 7. A. 8. A. 9. A. 10. A. 11. A. 12. A. catchy B. confidential C. fishy D. accidental Writing rhymes for birthday cards is really easy. Its money for old --boot B. rubbish C. bread D. rope

After many years as a doctor, he had become ---- to scenes of human distress. hardened B. experienced C. expert D. unemotional

The wheels ---- as the car went over an icy patch. skipped B. slid C. skidded D. slipped

The word processor --- of a keyboard, a monitor and a printer. composes B. consists C. comprises D. includes

Those men were appointed by the directors and are --- only to them. accountable B. dependable C. privileged D. controlled I hope you wont take ---- if I tell you the truth. annoyance B. offence C. resented D. irritation

They --- for Paris at midnight. set off B. made for C. went off D. got down Giuseppes secretary ---- a call to the office in Milan. applied for B. put through C. put by D. set about

81

HUNG VUONG SUMMER C AMP-2010

13. A. 14. A. 15. A. 16. A. 17. A. 18. A. 19. A. 20. A.

Ingrid broke ---- in tears when we told her about the accident. off B. through C. down D. for

I expect the new trend will soon ---- here. catch up B. catch on C. take up D. identify with Its difficult to tell identical twins --on B. out C. apart D. over Thank goodness you will be there. Im ---- on your support. holding B. calling C. hanging D. counting

The reporter announced solemnly that the president had ---- in his sleep. passed away B. died away C. passed out D. dropped off

Politicians frequently ---- a lot of criticism. come out B. catch up with C. come in for D. get up to No matter how often I explain it, he doesnt seem to --put it in B. take it in C. take it on D. put it through

Lucia was surprised when her guests --- late for the party. came up B. turned up C. looked up D. put up

2. Give the correct form of the word given. 21. Most peasants were living in subhuman conditions as the Feudalism had ------------

------- them for a long time. POVERTY 22. 23. Many ------------------- have been built since the end of the war. MEMORY As a result of his serious crime, he was sentenced to death by -------------------Its undeniable that the ------------------- of the local incompetent healer was I was ----------------- put off studying by the annoying noise from a neighbors

ELECTRICITY 24.

responsible for her sudden death. DIAGNOSE 25.

apartment last night. CONTINUE 26. The fact that ------------------- language conveys about 60% the information

received from a conversation is unbelievable to almost everybody. VERB 27. This school was once -------------------- as a military hospital during the war.

REQUIRE

82

HUNG VUONG SUMMER C AMP-2010

28.

The courts were ------------------ to impose the death penalty for certain crimes. He hasnt got the ------------------ capacity to think of such a clever plan. IMAGINE His recent ----------------- behavior keeps me wondering whether there is something

POWER 29. 30.

wrong going on. TYPE

3. Find 10 mistakes in the passage and correct them.

Because different tree species adapting to different climate and soil types have evolved over millennium, many kinds of forests occupy the earth today. The primitive forests of several hundred million years ago consisted of less kinds of trees. In fact, the earliest trees, which grew nearly 500 million years ago, were like giant club mosses. They lack true roots and consisted of mass specialized branches that climbed over rocky ground. 50 million year later came a dense forests of tree ferns that prevailed tropical climates of that era. The forerunners of modern conifers trees that bear cones were on the scene 300 million years ago, when plant life abundant colonized marshy land, building the tremendous coal and oil reserves so important today. By the time the dinosaurs roamed the earth some 180 million years before, seed bearing trees that shed their leaves in winter evolved; from these have sprung our present deciduous forests.

31.------------------32. -----------------33. -----------------34. -----------------35. -----------------36. -----------------37. -----------------38. -----------------39. -----------------40. ------------------

4. Give the correct form of the verbs. When the bus stopped in a small square, Helen (41) --------------- ( read) her magazine and didnt realize that she (42) -------------- (arrive) at her destination. This is Santa Teresa, Martin said You (43) -------------- (arrive) home! I suppose your cousin (44) ---------------- (wait) for us. Come on, I (45) -------------------- (carry) the bags. Helen thought, all those years when I (46) ----------------- (live) in New York, I dreamt of

83

HUNG VUONG SUMMER C AMP-2010

this moment. And now its real, I cant believe it! Here I am! I (47) ---------------- (really/ stand) in the square. Santa Teresa was Helens birthplace, but she (48) -----------------(leave) the town at the age of six. She had some memories of the town, and some photographs but did she belong her still? She didnt know. Nobody (49) -----------------(wait) in the square. Perhaps her cousin Maria (50) -------------------------- (not receive) Helens letter. What are we doing now? asked martin, there isnt even a hotel here!

5. Fill in each blank with a suitable article a, an, the or . Lead poisoning in children is (51) ------------- major health concern. Both low and high doses of paint can have serious effects. Children exposed to (52) ------------ high doses of lead often suffer (53) ----------- permanent nerve damage, mental retardation, blindness and even death. Low doses of lead can lead to (54) ----------- mild mental retardation, short attention spans, distractibility, poor academic performance, and behavioral problems. This is not (55) ------------ new concern. As early as 1904, lead poisoning in children was linked to lead based paint. Microscopic lead particles from paint are absorbed into (56) -------------- bloodstream when children ingest flakes of chipped paint, plaster, or paint dust from sanding. Lead can also enter (57) --------------- body through household dust, nail biting, thumb sucking or chewing on toys and other objects painted with lead based paint. Although American paint companies today must comply with strict regulations regarding (58) ------------ amount of lead used in their paint, this source of lead poisoning is still (59) ------------------ most common and most dangerous. Children living in order, dilapidated hoses are particularly at (60)----------- risk.

PART II. READING COMPREHENSION 1. Read the passage above again and circle the best ans wers to the questions. 61. What is the main topic of the passage? A. problems with household paint C. lead poisoning in children B. major health concerns for children D. lead paint in order homes

62. The phrase exposed to in paragraph 1 could be best replaced by A. familiar with B. in contact with C. displaying D. conducting 63. As used in paragraph 2, the word suffer is closest in meaning to

84

HUNG VUONG SUMMER C AMP-2010

A. experience

B. reveal

C. feel pain from

D. grieve with

64. Which of the following does the passage infer? A. paint companies can no longer use lead in their paint. B. paint companies arent required to limit the amount of lead used in their paint. C. paint companies must limit the amount of lead used in their paint. D. paint companies have always followed restrictions regarding the amount of lead in their paint. 65. Which of the following is closest in meaning to the phrase linked to in paragraph 2? A. endorsed by B. threatened by C. combined with D. associated with 66. The word absorbed in paragraph 2 could be best replaced by A. fixed B. assimilated C. soaked D. accepted

67. Where is the best place in the passage to add the following sentence? Older, run-down houses commonly have chipped, flaking paint and are more likely to have been painted with lead based paint. A. B. C. D. In line 1 after the phrase health concern In line 5 after the word problem. In line 6 after the words new concern In line 13 after the word risk.

68. Which of the following is closest in meaning to the word chipped as used in paragraph 2? A. fragmented B. canned C. sprayed D. unhealthy 69. The word dilapidated in paragraph 2 is closest in meaning to A. poorly painted B. fallen down C. unpainted D. broken down

70. According to the passage, what is the most common source of lead poisoning in children? A. household dust B. lead-based paint C. painted toys D. dilapidated houses

2. Read the following passage and choose from the list A G the sentences which best summarizes each part of the article. ---------- 71. Like technical difficulties, studio audiences are just another common feature of television life, and yet to many of us, they remain a mystery. Watching them getting

85

HUNG VUONG SUMMER C AMP-2010

excited on game shows, for example, we sit back and ask ourselves just who are these people? ---------- 72. Of all the audiences for live entertainment, the studio variety is widely considered to be the lowest of the low. I have heard it said that even people who work in television treat studio audiences with scorn or, as an cameraman put it, I like cattle. ---------- 73. I had assumed that studio audiences were made up of silly people desperate for two seconds of fame. But theres no such thing as a typical studio audience. They come from all classes, professions and income groups. Television tries to attract different types of people for different types of programs. ----------- 74. Those of us who prefer to watch television from home cant see why anyone would want to watch television from a studio. Why would anyone bother to apply for tickets, travel long distances, and suffer hours of boredom in the discomfort of a studio just to watch what they can see at home? ------------ 75. One theory is that people hope that for a second they might appear on television. I didnt believe this until I spoke to Angela. Why had she come? It was a chance to appear on TV. Another theory is that people are curious to take a look behind the scenes. But the most common explanation I heard was simply a case of a friend gave me a ticket. A. B. C. Studio audiences consist of all kinds of people. For some people, being in a studio audience is preferable to watching TV at home. It is hard for some people to understand the reasons for wanting to be in a studio

audience. D. E. F. G. Studio audiences play an important part in the making of TV programs. Members of studio audiences are sometimes not regarded with respect. People become part of a studio audience for various reasons. Viewers wonder what kind of people are in studio audiences.

VIII. Cloze test Health is something we tend to ignore when we have it. When our body is rather well, we are hardly (76) -------------- of it. But illness can come, even (77) ------------ we are young. In fact, childhood has been a very vulnerable time. Many (78) ---------------- attacked

86

HUNG VUONG SUMMER C AMP-2010

children in particular and people knew very little abo ut how to prevent such illness or how to (79) ---------- them once they struck. The result was that many children died. About a century ago, however, scientists found out about germs, and then everything (80) -------------- . The cause of many diseases was known, and cures were developed. As medical knowledge spread, the world became (81) -------------- safer for children. The result is that whereas a hundred years ago the average man lived for 35 years. Nowadays, in many (82) ----------------- of the world, people can expect to (83) --------------- for years. And what do we expect by the year 2050? Undoubtedly, (84) ------------------ science will continue to advance. Some people will be able to (85) ------------- medical problems that are unavoidable today.

IX.

Rewrite the following sentences.

86. Its only after a few weeks that you begin to feel at home here. You wont -------------------------------------------------------------------------------- --------------87. Hes almost certain to leave before we do. By the time ------------------------------------------------------------------------------------ ---------88. Lucas was last heard of a week ago. Nobody -------------------------------------------------------------------------------------------------89. Theo is the most infuriating person Ive ever met. Ive ------------------------------------------------------------------------------------------------------90. Never before have I seen Anita with her hair in such a mess. This is ---------------------------------------------------------------------------------------------------91. It appears that they sent us the wrong information. They -----------------------------------------------------------------------------------------------------92. The President clearly felt the ministers he sacked had not acted swiftly enough. The ministers sacked ----------------------------------------------------------------------------------93. Im glad I got out of there; It was hell. Im glad to ----------------------------------------------------------------------------------------------94. The accused became very emotional. OVERCOME ................................................................................................................................................. 95. Naturally, they will deduct points if you arrive late. PENALISED

87

HUNG VUONG SUMMER C AMP-2010

................................................................................................................................................. 96. The models clothes lay all over the floor. STREWN ................................................................................................................................................. 97. Our new Director wants you call her Madam. ADDRESSED ................................................................................................................................................. 98. I cant say I enjoy it when people tear my writing to pieces in front of me. HAVING ................................................................................................................................................. 99. We will have to make up our minds by the end of the week, wont we? BE ................................................................................................................................................. 100. Its just possible the hotel may need more staff in the summer. RULED .................................................................................................................................................

88

HUNG VUONG SUMMER C AMP-2010

TEST 5

PART I: VOCABULARY 1. Choose the best word or phrase to complete each sentence (1-15). Write A, B, C, or D in the space provided according to your choice. 1. Houses in big cities are expensive because land is in ............................ supply. B. slight C. little D. short

A. brief 2.

They ............................ the advantages and disadvantages of moving and finally

decided against it. A. meditated 3. Speed King. A. calls 4. B. has called C. called D. is calling B. considered C. thought D. wondered

At the races a man ............................... Taylor bought a lot of winning tickets on

In Venice the quickest way to get around is to take a boat along the

................................. A. canals 5. B. channels C. tracks D. trails

All her family came to the airport to ............................. her off on her trip to

London. A. call 6. B. get C. see D. take The news of her sisters death came as a(n) ........................... shock to him. B. entire C. extreme D. great

A. utter 7.

I quite .......................... Mozart, but on the whole I prefer Beethoven. B. love C. like D. idolize

A. adore 8.

When morning came, the scene of where the bomb had fallen was one of utter

........................... A. disturbance 9. B. damage C. disruption D. devastation

When we were taking the exam, our teacher said that honesty .............................

the best policy. A. is 10. B. was C. had been D. will be

Overwork is .............................. to cause increased stress. B. possible C. probably D. obviously

A. likely

89

HUNG VUONG SUMMER C AMP-2010

11.

Many young people travel all over the world, and do all kinds of jobs before they

......................... A. lie down 12. B. settle down C. touch down D. put down

One-day ........................... to Oxford and Stratford are very popular with tourists

visiting London. A. expeditions 13. B. travels C. excursions D. voyages

In the old days it was common to see a farmer driving a horse and

................................. A. cart 14. B. tractor C. truck D. van

400 workers are facing ............................... because the factory is in financial

difficulty. A. employment 15. B. unemployment C. jobless D. unworked It is just as well you didnt put your money in that company, because it would

have been an absolute ..................................... if you had. A. catastrophe B. mistake C. error D. miscalculation

Write your answers in the following space. 1. 6. 11. 2. 7. 12. 3. 8. 13. 4. 9. 14. 5. 10. 15.

PART II. USE OF ENGLISH 1. Think of one suitable word to fill in the numbere d blanks (16-30). Write your ans wers to the space provided under the text. Mozart, who (16) ............. born on January 27 th 1756 in the Austrian city of Salzburg, was neither the first nor the last child prodigy, (17) ...... ...... he was certainly the greatest. From the age of six, (18) ................ his father took him on his first foreign tour, Mozart toured the courts (19) ............ musical centers of Austria, Germany, France, England, Holland, Switzerland and Italy. It has been calculated that Mozart (20) .................... almost a third of his short life he died (21) ...........the age of 35 traveling.

90

HUNG VUONG SUMMER C AMP-2010

Mozart was born (22) ........... a moderately prosperous family (23) ........... his musical genius made itself known extremely early. Mozart (24) .................. learning harpsichord at three and his early known work was (25) ............... in 1761 when he was five, the age at (26) .............. he also first appeared in public. At seven, never having had a violin lesson, he picked up the instrument and played (27) ............... perfectly. Mozart and his older sister Nannaerl were the only survivors of seven children. Both were exceptional musicians and their father Leopord, who was himself a noted violin teacher, took them on several tours. Mozart lived in England (28) ........... April 1764 until August 1765, mainly in Chelsea, giving concerts and composing. It was here, probably, (29) ............ he wrote the first of his symphonies. Mozart never revisited England, (30) ........... he frequently planned to return.

Write your answers in the following space 16. 17. 18. 19. 20.

21.

22.

23.

24.

25.

26.

27.

28.

29.

30.

PART III: SITUATIONAL USE OF ENGLISH Catherine is talking to an archaeologist, Dr Blake, about some ancient jewelry he has recently dug up. Complete the conversation. Catherine: Tell me about this object. Its very decorative, but (31) ......................? Dr Blake: Not really, it was just a personal ornament. Its only function was to show the status of the wearer, a bit like diamonds show status today. Catharine: What (32) ............................................? Dr Blake: Gold, and these beautiful red tones are garnets. Catharine: How (33) ....................? Dr Blake: Like a brooch, probably pinned on the shoulder.

91

HUNG VUONG SUMMER C AMP-2010

Catharine: It looks very modern. Could you tell me when (34) ........................? Dr Blake: Probably at the end of the seventh century. Catharine: (35) ...............................? Dr Blake: In a womans grave in south-east England. Catharine: So that woman must have owned the brooch. What sort (36) ............................? Dr Blake: Well, it cant have been an uncomfortable one, because she must have been quite wealthy. Ill know more about her life when Ive dug up more of the site. Catharine: The brooch doesnt look as if its been buried in the earth for hundreds of years. Was (38) ................................? Dr Blake: No, not at all. Weve had to do a lot of cleaning and restoration work on it. Catharine: Its certainly a beautiful piece. Id love to wear it! Thank you for showing it to me.

PART IV: READING COMPREHENSION 1. Read three short texts, then ans wer the questions (39 - 63) following each text by choosing the best answer to each question, A, B, C, or D. Then write the letter A, B, C, or D indicating your choice in the space provided.

A.

William Faulkner of Oxford was not a native of Oxford; nor was he born with the name Faulkner. He was born in New Albany, Mississippi, on September 25 th , 1897 and the family spelled the name Falkner. He published his first book when he was twenty-seven. He was awarded the Nobel Prize for literature when he was fifty- three. He was generally acknowledged as the major American writer of his time. When he died on July 6 th 1962, Faulkner spent almost the whole of his life in the Mississippi town. Even to the people of Oxford, Faulkner was a kind of legend in his own lifetime. There was for example the mystery of who put the letter u in the Williams last name. For many years, the commonly accepted story was that it was a careless printer, in setting type of the Marble Faun (1924). Faulkner biographer Carvel Collins has demonstrated that the writer himself added it, and at least occasionally, as early as 1918.

92

HUNG VUONG SUMMER C AMP-2010

39. At the time of his death, Faulkner was .................................. A. 27 B. 53 C. 62 D. 65

40. He spent most of his life in ................................ A. Oxford B. a foreign country C. his hometown D. London

41. He was awarded the Nobel Prize in .................................. A. 1897 B. 1924 C. 1950 D. 1962 42. The letter u in Faulkners name was added by ............................... A. Faulkner himself C. Faulkners biographer Carvel Collins B. a careless printer D. Faulkners family

43. He was acknowledged as ........................................... A. an Oxford citizen. B. the major American writer of his time. C. the only writer that got the Nobel Prizes of Literature. D. all are correct.

B. Louis Braille was born in 1809 in a small town in France. His father had a small business. He made shoes and other things from leather. Louis liked to help his father in the store. One day when Louis was three years old, he was cutting some leather. Suddenly the knife slipped and hit him in the eye. Soon Louis could not see at all. At first Louis went to school in his village with other children who were not blind. He understood immediately that it was very important fo r people to learn how to read. He also understood that blind people needed a special way to read. When he was ten years old, Louis went to Paris to study at the National Institute for the Blind, a special school for blind students. There he found huge books with large, raised letters. Students tried to read by feeling the big letters. Louis decided to find a better way for blind people to read. One day Louis went with his class to a special exhibit by a captain in the army. One thing in this exhibit was very interesting to Louis. The captain showed him a special code he had made. The army used this code to send messages at night. People in the army wrote this code in raised marks on very thick paper.

93

HUNG VUONG SUMMER C AMP-2010

Louis thought a lot about this code. Then he decided to write his own code so blind people could read with their fingers. Louis found that it was very difficult to feel the difference between raised letters. Instead of letters, Louis used a system of raised dots. He arranged them in a cell of six dots, with two dots across and three down. With his cell of dots, Louis could make sixty-three different combinations. Each combination stands for one letter, punctuation mark, or number. Louis invented this system when he was only fifteen years old. It is called the Braille system. Blind people can read by feeling the raised dots with their fingers. They know what letters the dots represent. Today Braille books are found in all written languages in the world. In many countries you can find signs in Braille in elevators and in buildings such as hospitals. People can also write in Braille. There are special pens, typewriters, and computers for Braille. Louis Braille invented a way for blind people to communicate through reading and writing. When he died in 1852, he was buried in the Pantheon in Paris. Only national heroes of France are buried in this place, and Louis Braille was a national hero. 44. What was Louis Brailles nationality? A. France B. England C. French D. English

45. How old was Louis Braille when the accident happened? A. 1 B. 3 C. 5 D. 10

46. When Louis was ten years old, he began to study ................................ A. at a university C. at his neighborhood school 47. Which of the following is NOT true? A. Louis lost his eyesight because he was hit in the eye by a knife when he was helping his father. B. Louis invented the Braille system when he was fifteen years old. C. At the class, Louis discovered a code the army used to send messages at night. D. Feeling the difference between raised letters was very difficult. 48. The phrase stand for means most nearly the same as ......................... A. represent B. arrange C. combine D. call B. in the army D. at a special school for the blind

94

HUNG VUONG SUMMER C AMP-2010

49. The word They refers to .............................. A. the feelings B. the blind C. their fingers D. the raised dots 50. The word combination is closest in meaning to ................................. A. dot B. cell C. addition D. arrangement

51. What is the main idea of this passage? A. Louis Braille visited an exhibit of codes used in the army. B. Louis Braille invented a system of reading for visually impaired people. C. Louis Braille didnt like the raised letters. D. Louis Braille was buried in the Parthenon, Paris.

Write your answers in the following space 44 48 45 49 46 50 47 51

C. In 1831, a young student of botany named Charles Darwin accepted an invitation to act as a traveling companion to a sea captain on a three-year scientific expedition. Nearly four years later, in 1835, the ship came to a group islands about 400 miles west of South America called the Galapagos archipelago. The islands had long been a hideout for pirates and were known as a popular hunting ground for whalers and seal hunters. But to Darwins highly trained eye, the islands were a unique and fascinating opportunity. During his five weeks on the islands, he observed a remarkable variety of flora and fauna as well as the many interesting non-living, geological aspects of the islands. Each island seemed to have slightly different variations of the same animals and plants. The particular features of these animals and plants were perfectly suited to the environment of their respective islands. His experiences on the Galapagos Islands would later bring Darwin to write his famous work On the Origin of Species, in which he proposed the ideas that led to the theory of evolution. Today, the Galapagos Islands are a protected natural park under the Ecuadorian government, and they are a UNESCO world heritage site. They are perhaps best known

95

HUNG VUONG SUMMER C AMP-2010

for their 14 subspecies of giant tortoise, which can weigh up to 250 kilograms and live for over 100 years. The islands actually take their from the tortoises: galapagos is the Spanish word for these tortoises, perhaps coming from a word meaning saddle. Scientists theorize that normal-sized tortoises originally floated to the islands long ago on water currents from mainland South America. A) They grew to their enormous size because the islands lacked any kind of natural predators. B) It is estimated that there were originally about 100,000 tortoises living in Galapagos. C) The tortoises can go for months without food or water. D) Unfortunately, this made them an attractive source of food for pirates and whalers in the 18 th and 19th centuries because the tortoises were simple to capture, they provided a lot of meat, and they could be stored live on ships over long voyages. Today there are only about 15,000 giant tortoises left alive. Three of the fourteen subspecies are now completely extinct. A fourth subspecies has only one single member left, who is known as lonesome George. Efforts to find a female mate for George have thus far been unsuccessful. When George dies, it will likely mark the extinction of a fourth species. George lives at the Charles Darwin Research Station on Santa Cruz Island, an environmental preservation center established in 1959. One of their major goals is to safeguard the populations of giant tortoises on the islands. Many populations are still under threat from foreign animals that were introduced to the islands by the Europeans. Dogs, rats, and pigs eat their eggs and young. Larger animals, such as donkeys, cattle, and goats step on their nests and eat their food sources. There are even still instances of humans killing the tortoises: roughly 120 tortoises have been poached from the islands of Isabella since 1990. To combat this, the Research Station runs a population program. Eggs are taken from the wild and hatched at the Research Station. The young are released back into the wild after they are big enough to survive attacks from other animals. The program has enjoyed some success and has safeguarded the populations of several of the islands. However, the future of the Galapagos Islands and its unique forms of life is far from certain. Non-native animal species and human activities, such as fishing, continue to threaten this once isolated ecosystem.

96

HUNG VUONG SUMMER C AMP-2010

52. Which of the following could best replace the word ground as used in paragraph 1? A. Dry land B. Area C. Floor D. Soil 53. The expression flora and fauna in paragraph 2 is closest in meaning to .................... A. hills and B. rivers and C. birds and D. plants and

mountains

lakes

mammals

animals

54. The word that in paragraph 6 refers to .................................. A. the animals introduced by Europeans C. the threat to the tortoises B. the Galapagos Islands D. the populations of tortoises

55. Why does the author mention that there were originally 100,000 tortoises? A. To emphasize the surprising number that led to Darwins theory. B. To contrast it with the small number alive today. C. To point out the breeding capabilities of the tortoises. D. To indicate their high value as a food source. 56. How long was Charles Darwin a visitor on the Galapagos Islands? A. About 4 months B. 12 days C. 5 weeks D. Over 3 years

57. According to the passage, why were tortoises hunted by pirates and whalers? A. Because they lasted a long time on boats. B. Because their meat was prized as a delicacy. C. Because they were exciting to hunt. D. Because the pirates used their shells as shields. 58. Which predators still eat the giant tortoises? A. Dogs, cats, wolves, and seals C. Pigs, goats, seals, and cats B. Humans, rats, goats, and hawks D. Dogs, rats, humans, and pigs

59. What does the last paragraph in the passage mainly discuss? A. Efforts to protect the giant tortoises. C. Tourism and giant tortoises. B. Dangers to the giant tortoises. D. The mating habits of the giant tortoises.

60. Look at the four squares () that indicate where the following sentence could be added to paragraph 4: Also larger male tortoises usually win in the competition for female mates.

97

HUNG VUONG SUMMER C AMP-2010

Where would the sentence best fit? Choose the square () where the sentence should be added to the passage. (A) Line 2 (B) Line 3 (C) Line 4 (D) Line 5 61. Based on the information in paragraph 1, what can be inferred about the ships voyage? A. The ships mission was to chart the Pacific Ocean. B. The voyage lasted longer than was originally planned. C. The ship was attacked by pirates on the voyage. D. Charles Darwin was a famous scientist at the time. 62. According to the passage, which is NOT true? A. The Spanish used the tortoise shells to make saddles. B. Horses sometimes step on the nests of giant tortoises. C. Darwin based his theories on his observations in Galapagos. D. There is only one survivor of the fourth tortoise subspecies. 63. Directions: An introductory sentence for a brief summary of the passage is provided below. Complete the summary by selecting the THREE answer choices that express the most important ideas in the passage. Some sentences do not belong in the summary because they express ideas that are not presented in the passage or are minor ideas in the passage. First Sentence: Charles Darwin, upon visiting the Galapagos islands in 1831, observed the slight differences in species native to the area and came up with the idea of evolution.

A. B.

Pirates had long used the archipelago as a hideout. These days, the Galapagos are a government-protected natural park, ensuring the

survival of many unique species including the giant tortoise. C. D. These giant tortoises can weigh as much as 250 kilograms. Scientists estimate that more than 100,000 tortoises lived in the Galapagos at the

time of Darwins arrival. E. In the years since Darwin arrived on the island, the population of the giant tortoises

has decreased dramatically.

98

HUNG VUONG SUMMER C AMP-2010

F.

To help protect the animals, the Charles Darwin Research Station runs a

repopulation program.

PART VI: SENTENCE TRANSFORMATION 1. Finish the second sentence in such a way that it means exactly the same as the sentence printed before it. 64. Silver is half the price of gold here. Gold costs ............................................................................................................................... 65. John didnt apply for that job until last year. It was not ................................................................................................................................ 66. The farmer went to bed and at once felt asleep. No sooner ............................................................................................................................. 67. Andrew is the most generous person I have ever met. Ive yet .................................................................................................................................... 68. Im sorry now that I asked her to stay. I wish ...................................................................................................................................... 69. The only reason the party was a success was that a famous film star attended. Had it not ............................................................................................................................... 70. I would love to live in London for a year. If only ....................................................................................................................................

2. Write a ne w sentence similar in meaning to the one given, using the word given. Do not alter the word in any way. 71. Bad weather delayed the flight. DUE 72. My jewelry has been stolen! OFF 73. The result of the match was a deep disappointment to the fans.

99

HUNG VUONG SUMMER C AMP-2010

DEEPLY 74. They share a lot of hobbies and interests. COMMON . . 75. From the educational point of view his childhood years had been well spent. TERMS 76. Dont eat so many sweets and you wont have to visit the dentist so often. SAVE 77. There were no volunteers for the school concert. TAKE 78. When I saw Jim it suddenly dawned on me how much he must have suffered. HOME

100

HUNG VUONG SUMMER C AMP-2010

TEST 6

PART I: VOCABULARY AND GRAMMAR 1. Choose the word or phrase which best completes each sentence (10 points). 1. The goods were displayed so well that I couldnt ______ the temptation to buy. A. pull B. deny C. resist D. miss

2. His ______ of the plane was so correct in every detail and could really fly. A. model B. design C. pattern D. shape

3. At parties, people ______ to talk about jobs and money. A. accustom B. habit C. tend D. keep

4. Because of bad weather, I propose that the meeting ______. A. is postponed A. mustnt go B. to be postponed C. be postponed D. postpones

5. He has just left here and ______ far. B. couldnt go C. cant have gone D. cant be going 6. She wasnt paying attention and crashed ______ a parked car. A. with B. down C. over D. into 7. They took ______ his passport so that he wouldnt leave the country. A. up B. away C. out D. off

8. At first, the children enjoyed the game but their interest soon ______ . A. went off B. died out C. died down D. wore off 9. I usually buy my clothes ______. Its cheaper than going to the dressmaker. A. on the house B. off the peg C. in public D. on the shelf

10. My father ______ when he found out that I had damaged his car. A. hit the roof C. made my blood boil Your answers: 1.. 6.. 2. 7. 3 8 4 9 5 10.. B. saw pink elephants D. brought the house down

2. Supply the correct tense of the verbs in brackets (10 points).

101

HUNG VUONG SUMMER C AMP-2010

After a dinner (1. give) ______ by the President of an African nation, a (2. distinguish) ______ diplomat complained to his host that the Minister of Justice, who (3. sit) ______ on his left, (4. steal) ______ his gold watch. "He (5. not/do) ______ that " said the President. "I (6. get) ______ it back for you". Surely enough, towards the end of the evening the watch (7. return) ______ to its owner. "And what he (8. say) ______?" asked the diplomat. "Shh!" said the host, (9. glance) ______ anxiously at him. "He doesnt know that I (10. get) ______ it back." Your answers: 1. 4. 7 10 2. 5. 8 3.. 6 9.

3. Fill in each blank with the correct form of the words in brackets (10 points). According to some scientists, high-risk sports can be particularly (1. VALUE) ______ for certain types of people. Such activities help them to learn that being (2. FRIGHT) ______ doesnt mean that they have to lose control. The recent fashion for jumping from bridges attached to a (3. LONG) ______ of an elastic rope known as bungee jumping has now been tried by over one million people (4. WORLD) ______ and interests in it continue to grow. Before the elastic rope (5. TIGHT) ______ around them, jumpers reach the speed of nearly 160 kph. First-timers are usually too (6. TERRIFY) ______ to open their mouths and when they are finally (7. LOW) ______ safely to the ground, they walk with broad smiles on their faces, saying (8. REPEAT) ______ how amazing it was. However, for some people, it is only the (9. EMBARRASS) ______ of refusing to jump at the last minute that finally persuades them to conquer their fear of (10. HIGH) ______ and push themselves off into space. Your answers: 1. 4. 7 10 2. 5. 8 3.. 6. 9.

102

HUNG VUONG SUMMER C AMP-2010

4. There are ten unnecessary words in most lines of the following passage. They are either grammatically incorrect or dont fit in with the sense of the passage. Underline each and write it in the space next to the line numbe r. Put a tick ( correct lines. (10 points) Wanted ran a small ad in The Times Assistant for the famous cookery writer. Three- month contract - 400 USD . The ad was answered by a young woman, recently widowed and with a small baby, desperate for to work of any kind. The hours were long and 400 USD seemed very little for three months of employment. But she was desperate and she got the job. It proved harder than she had been expected as the writer proved as tyrannical, ungrateful and a slave-driver. The first week of helping him was almost more than the young woman could stand. Only did the thought of the bread that she was putting in her babys mouth prevented her from leaving for her job. At the end of the first terrible week, she was given a lift to home by the writer s 7 secretary. On the way home, she confided how desperately she had needed the job and admitted how welcome it would be even the miserable 400 USD was mentioned in the advertisement. The secretary gave her such an odd look that the young 10 woman asked what it the matter was. I dont think you quite understand replied the secretary. Its you who have to pay for him 400 USD 12 11 9 8 6 5 4 3 2 1 the ) next to the

103

HUNG VUONG SUMMER C AMP-2010

PART II: READING COMPREHENSION 1. Read the passage carefully and then choose the best answer to fill in each blank (10 points). Clean freshwater resources are essential for drinking, bathing, cooking, irrigation, industry, and for plant and animal (1) ____. Unfortunately, the global supply of freshwater is (2) ____ unevenly. Chronic water shortages (3) ____ in most of Africa and drought is common over much of the globe. The (4) ____ of most freshwater supplies - groundwater (water located below the soil surface), reservoirs, and rivers - are under severe and (5) ____ environmental stress because of overuse, water pollution, and ecosystem degradation. Over 95 percent of urban sewage in (6) ____ countries is (7) ____ untreated into surface waters such as rivers and harbors; About 65 percent of the global freshwater supply is used in (8) ____ and 25 percent is used in industry. Freshwater (9) ____ therefore requires a reduction in wasteful practices like (10) ____ irrigation, reforms in agriculture and. industry, and strict pollution controls worldwide.

1. 2. 3. 4. 5. 6. 7. 8. 9. 10.

A. survive A. delivered A. exist A. resources A. increasing A. growing A. recharged A. farming A. reservation A. ineffective 1.. 6..

B. survived B. distributed B. lie B. springs B. growing B. miserable B. discharged B. planting B. conservation B. illogical 2. 7. 3 8

C. surviving C. provided C. show C. sources C. climbing C. poverty C. charged C. agriculture C. preservation C. irrational 4 9

D. survival D. given D. stay D. starting D. ascending D. developing D. discharging D. growing D. retention D. inefficient 5 10..

Your answers:

104

HUNG VUONG SUMMER C AMP-2010

2. Read the passage carefully and choose the best answers (10 points). In the United States, presidential elections are held in years evenly divisible by four (1888, 1900, 1964, etc.). Since 1840, American presidents elected in years ending with zero have died in office, with one exception. William H. Harrison, the man who served the shortest term, died of pneumonia only several weeks after his inauguration. Abraham Lincoln was one of four presidents who were assassinated. He was elected in 1860, and his untimely death came just five years later. James A. Garfield, a former Union army general from Ohio, was shot during his first year in office (1881) by a man to whom he wouldn't give a job. While in his second term of office (1901), William McKinley, another Ohioan, attended the Pan-American Exposition in Buffalo, New York. During the reception, he was assassinated while shaking hands with some of the guests. John F. Kennedy was assassinated in 1963 in Dallas only three years after his election. Three years after his election in 1920, Warren G, Harding died in office. Although it was never proved, many believe he was poisoned. Franklin D. Roosevelt was elected four times (1932, 1936, 1940 and 1944), the only man to serve so long a term. He had contracted polio in 1921 and eventually died of the illness in 1945. Ronald Reagan, who was elected in 1980 and reelected four years later, suffered an assassination attempt but did not succumb to the assassin's bullets. He was the first to break the long chain of unfortunate events. Will the candidate in the election of 2020 also be as lucky? 1. All of the following were election years EXCEPT _________ A. 1960 2. B. 1930 C.1888 D.1824

Which president served the shortest term in office? A. Abraham Lincoln C. William McKinley B. Warren G. Harding D. William H. Harrison

3.

Which of the following is true? A. All presidents elected in years ending in zero have died in office. B. Only presidents from Ohio have died in office. C. Franklin D. Roosevelt completed four terms as president. D. Four American presidents have been assassinated.

105

HUNG VUONG SUMMER C AMP-2010

4. office?

How many presidents elected in years ending in zero since 1840 have died in

A. 7 5.

B. 5

C. 4

D. 3

The word "inauguration" in the first paragraph means most nearly the same as

_________ A. election campaign 6. All of the following presidents were assassinated EXCEPT _________ B. Franklin D. Roosevelt C. Abraham Lincoln D. James B. acceptance speech C. swearing- in ceremony D.

A. John F. Kennedy 7.

The word "whom" in the second paragraph refers to _________ A. Garfield B. Garfield's assassin C. a Union army general D. McKinley

8.

The word "assassinated" in the second paragraph is closest in meaning to

_________ A. murdered 9. B. decorated C. honored D. sickened

In the third paragraph, "contracted" is closest in meaning to _________ A. communicated about B. developed C. agree about D. notified

10. How long did Warren G, Harding work as a president? A. 2 years Your answers: 1.. 6.. 2. 7. 3 8 4 9 5 10.. B. 3 years C. 4 years D. 4 years

3. Read the following passage and use ONE word to fill in each gap (10 points). Are you looking ___1___ a cheap, clean, effective source of power that doesn't ___2___ pollution or waste natural resources? Look no further than solar ___3___ . While most fuels now in use are being burned ___4___ an astonishing rate, solar energy or power from the ___5___ will last as long as the world ___6___ . This energy has already been used to heat and cool homes and to cook ___7___. It has been used experimentally ___8___ radio batteries and furnishing power for telephone lines. The devices, ___9___, are cheap to operate but very ___10___ to produce. Your answers:

106

HUNG VUONG SUMMER C AMP-2010

1.. 6..

2. 7.

3 8

4 9

5 10..

PART III: WRITING 1. Rewrite these sentences so that the ne w one has a similar meaning as the given one (5 points). 1. Children can learn a lot about how to behave in a situation like this. Only. 2. Without your help, I couldnt have finished that difficult task. If you 3. The storm completely wiped out all my crops. I had 4. The oil price has fallen considerably in the past months. There 5. Nobody expected her to lose but she did. Against 2. Rewrite each of the sentences using the given word so that the new one has a similar meaning as the given one. You must not change the given word (5 points). 1. She worked very hard to earn just enough money to pay her bills. (ENDS) . 2. You should do your homework now. (TIME) 3. The exam was very easy and I made no mistake. (CAKE) 4. Everyone heard about the accident before I did. (LAST) 5. I dont mind whether we have a meeting today or tomorrow. (MAKES) 3. Write a composition of about 200 words to ans wer the question:

"How does the Internet help you with your study?" (20 points)

107

HUNG VUONG SUMMER C AMP-2010

TEST 7

PART I: VOCABULARY AND GRAMMAR (15 points) 1. Choose the best option A, B, C, or D to complete the following sentences 1. The police were baffled by the attack as there seemed to be no apparent A. design 2. . A. form 3. B. format C. formation D. formality This new glue is very useful for small repairs as it very rapidly B. stiffens C. sets D. fixes B. principle C. motive D. plot Youll have to go for an interview tomorrow, but dont worry. Its just a

A. thickens 4. . A. taper off 5.

Demand for the product is expected to peak five years from now and then to

B.

fall down

C. gripped

D. distracted

The tour guide had a brightly-colored company badge pinned to the of

her jacket. A. lapel 6. B. border C. edge D. hem This cloth . very thin. B. touches C. holds D. handles I would appreciate .it a secret. A. your keeping B. you to keep C. that you keep D. that you will keep .team sports require cooperation. B. They are all surgeon Ephraim C. Why are all MacDonald had to perform D. All operations

A. feels 7. A. 8.

A. Of all 9. Frontier

.. anesthesia. A. no 10. B. not having C. there wasnt D. without The twins look so ..their father. B. likely C. likeness D. alike He decided to make a claim ..damages to his car.

A. like 11.

108

HUNG VUONG SUMMER C AMP-2010

A. for 12.

B.

on

C.

about

D.

in

We lay in the sun until our bodies felt saturated .. the heat. B. with C. upon D. within He by pointing out the dangers involved in rock

A. up 13.

climbing. A. started on 14. B. started off C. started up D. started out She ..till the early hours listening to pop music. B. caught me up C. kept me up D. took me up Im so tired that I cant take .what you are saying. B. out C. in D. on

A. held me up 15.

A. up

PART II: USE OF ENGLISH (15 points) 1. Fill in each of numbered blanks in the following passage. Use only one word in each space.

An Environmental Disater The huge oil spill in the (1) of Mexico, off the southern coast of the United States, has (2) in the news all around the world. Oil began to leak into the sea after an explosion on 20 th April on an oil (3)that was drilling for oil under the sea bed to the south of the coast of the state of Louisiana. Eleven workers on the oil rig were killed in the explosion. The rig caught fire and (4) to the bottom of the sea two days (5).. Since the accident (6).., around 800,000 liters of oil have been (7). into the sea every day from the broken oil well, which is on the sea bed about 1,500 meters below where the rig used to be. The company in (8)..of the rig is British Petroleum (BP). President Obama says BP must stop the leak and pay for the (9).. operation, although the US government will also give all the help it can. An oil spill of this size is usually an environmental disaster. The oil slick already measures more than 5,000 square kilometers, and some of the oil has already reached the Louisiana coast. (10). on how the slick moves, it could cause very bad pollution along the coast and have serious

109

HUNG VUONG SUMMER C AMP-2010

consequences for the fishing and tourism industries. It could a lso kill a huge number of seabirds and other (11).. There are now two very important operations in (12) one under the sea, and one on the surface. To stop more oil going into the sea, BP is trying to put a large metal container around the broken well. This is very difficult, however, because the well is so far underwater, and it could take many weeks to stop the leak completely. Meanwhile, to try to stop most of the oil reaching the coast, floating barriers have been (13).. on the surface of the sea, and some of the oil has been destroyed by setting (14).. to it. The accident has already had political consequences. President Obama previously agreed that there should be more offshore oil wells, (15).. means oil wells in the ocean near the coast, even though many people said the risk of pollution was too big. Since the accident, however, he has said there should be no new offshore drilling, at least until there has been an investigation to find out exactly how the explosion on 20th April happened. 1 6 27 38 4 9 510 11 12. 13 14 15

PART III: MAKING SENSE OF WHAT THE SPEAKER SAYS (5 points) 1. I like talking to people who have the sense of humor. A. They are people who have the sense of humor to that I like talking. B. It is talking to people that I like who have are sense of humor. C. It is people with the sense of humor that I like talking to. D. People who have the sense of humor are that I like talking to. 2. He is keen on collecting antique things. A. He is interesting in collecting antique things B. It is antique things that he is interested in collecting C. Collecting antique things is that he keen on D. It is collecting antique things that he is fed up with 3. Tomorrow we will go to the zoo or we visit the citadel because we dont have much time

110

HUNG VUONG SUMMER C AMP-2010

A. Tomorrow we will either go to the zoo or visit the citadel because we dont have much time. B. Tomorrow we will neither go to the zoo nor visit the citadel because we dont have much time. C. Tomorrow we will go to the zoo but we neither visit the citadel because we dont have much time. D. Tomorrow we will both go to the zoo and visit the citadel because we dont have much time. 4. "I will let you know the answer by the end of this week, Tom said to Janet. A. Tom offered to give Janet the answer by the end of the week. B. Tom suggested giving Janet the answer by the end of the week. C. Tom insisted on letting Janet know the answer by the end of the week. D. Tom promised to give Janet the answer by the end of the week. 5. My friend told me, If I were you, I would not drink so much. A. My friend advised me not to drink so much. B. My friend suggested not drink so much. C. My friend prohibited me from drinking so much. D. My friend warned me against drinking so much.

PART IV: SITUATIONAL USE OF ENGLISH (5 points) This is the conversation between a shop assistant and a woman who is interested in some scarves. Read and complete the conversation. Shop Assistant: Hi, (1)? W: No, Im not. (2)... Shop Assistant: All our scarves are in this section. What do you think of this one here? Its made of silk. B: Hm, it looks nice, but Id like to have something warm for the winter. A: Maybe you would like a heavy wool scarf. (3)..? B: I think thats what I want. (4)? A: Itsseventy- five dollars plus tax. B: Its a little expensive. Do you think its possible to get a discount? A: Hm, since you like it so much, how about a 10 percent discount. Thats the best I can offer.

111

HUNG VUONG SUMMER C AMP-2010

B: Thats good. Could you wrap it up for me? A: Sure. Is there anything else I can get for you? B: No, that should be it. (5)

PART V: READING COMPREHENSION ( 30 points) 1. Read the texts below and asnwe r the questios by choosing the best ans wer, A, B, C, or D.

A. Why does English spelling have a reputation for being difficult? English was firs t written down when Christian monks came to England in Anglo-Saxon (1). They used the 23 letters of Latin to write down the sound of Anglo-Saxon (2) as they heard it. However, English has a (3) .range of basic sounds (over 40) than Latin. The alphabet was too small, and so combinations of letters were needed to express the different sounds. Inevitably, there were inconsistencies in the way that letters were combined. With the Norman invasion of England, the English language was put (4) .risk. English survived, but the spelling of many English words changed to follow French (5)., and many French words were introduced into the language. The result was more irregularity. When the printing press was (6)in the fifteenth century, many early printers of English texts spoke other first languages. They made little effort to respect English spelling. Although one of the short-term (7) .. of printing was to produce a number of variant spellings, in the long term it created fixed spellings. People became used to seeing words spelt in the same way. Rules were drawn up, and dictionaries were put together which printers and writers could (8) to. However, spoken English was not fixed and continued to change slowly-just as it still does now. Letters that were sounded in the Anglo-Saxon period, like the k in knife, now became (9).. Also, the pronunciation of vowels then had little in common with how they sound now, but the way they are spelt hasnt changed. No (10) , then, that it is often difficult to see the link between sound and spelling.

112

HUNG VUONG SUMMER C AMP-2010

1. A. times 2. A. chat 3. A. wider 4. A. in 5. A. guides 6. A. discovered 7. A. actions 8. A check 9. A dumb 10. A. idea

.B. centuries B. communication B. longer B. at B. plans B. made up B. effects B. refer B. quiet B. wonder

C. ages C. speech C. thicker C. on C. patterns C. invented C. conclusions C. look C. speechless C. problem

D. years D. discussion D. deeper D. under D. types D. taken in D. meanings D. see D. silent D.mention

B. When Daniel Boone died peacefully in bed in his son Nathan's elegant stone Missouri farmhouse on September 26, 1820, the surge of emigrants along the Oregon Trail was still a generation away. But Boone already exemplified the pioneer at his best. He was neither the physical giant (five feet nine) nor the innocent child of nature that legend has made of him. He was an intelligent, soft spoken family man who cherished the same wife for 57 years. He befriended Indians, preferred company to solitude, and when he told his wife it was time to move because a newcomer had settled some 70 miles away, he was joking. Pennsylvaniaborn, Boone was one of 11 children in a family of Quakers who migrated to North Carolina. There Boone was recruited at age 40 to undertake a scheme designed to open up Kentucky to settlers and establish it as a 14th colony. He arranged a deal by which the Cherokees sold 20 million acres for $20,000 worth of goods to Boone's employers, the Transylvania Company. It was all fair and square-the Indians had an attorney, an interpreter, and the sound advice of their squaws. The deal completed, Boone led a party from Tennessee through the Cumberland Gap, hacked out the Wilderness Road, and set up a town- Boonsboro-and a government. Elected a legislator, he introduced on the first session's first day a bill to protect game against wanton slaughter and a second bill to "improve the breed of horses." He got 2,000 acres for his work, but after the Revolution- in which Boone won considerable fame as a militia commander-the scheme of the Transylvania Company was declared illegal and Boone lost his land. Undaunted, he staked out more claims-and lost them because he impatiently neglected to register his deeds. Ever hopeful, he accepted an invitation from Spanish-held Missouri to come and settle there and bring others with him. The Spanish gave him 8,500 acres and made

113

HUNG VUONG SUMMER C AMP-2010

him a judge. But the Louisiana Purchase, which embraced Missouri, again left him-but not his children- landless. Old and broke, Boone cheerfully continued hunting and trapping long after his hands shook. Shortly before he died, he was talking knowledgeably with young men about the joys to be experienced in settling California.

1. What is the author's purpose in writing this passage? (A) To chronicle the life of a model pioneer (B) To romanticize the legend of Daniel Boone (C) To show Boone's many successes on the frontier (D) To trace Boone's explorations in Kentucky, Missouri, and Louisiana 2. The word "surge" in the passage is closest in meaning to (A) remains (B) attraction (C) rush (D) rumor

3. It can be inferred that one area in which Boone was NOT successful was (A) politics (B) hunting and trapping (C) business (D) the military

4. The phrase "fair and square" in the passage is closest in meaning to (A) honest (B) simple 5. It can be inferred from the passage that Boone died (A) a rich man (B) an eternal optimist (C) in California (D) a lonely trapper (C) efficient (D) lucrative

6. According to the passage, where is Boone's namesake city located? (A) In North Carolina (B) In Transylvania 7. The Transylvania Company wanted Boone to (A) settle Kentucky (B) be fair to the Indians (C) ensure animal rights (D) claim Missouri (C) In Kentucky (D) In Missouri

8. The word "undaunted" in the passage is closest in meaning to (A) unscrupulous (B) fearless 9. According to the passage, the Louisiana Purchase (A) legitimized Boone's land claim in Missouri (C) undiscouraged (D) uninformed

114

HUNG VUONG SUMMER C AMP-2010

(B) revoked the earlier Spanish bequest to Boone (C) drove the Spanish from the East (D) excluded Missouri from its jurisdiction 10. What can be inferred from the passage about Boone's children? (A) They were better off financially than Boone. (B) They supported Boone's desire to settle new areas. (C) They lived in Kentucky. (D) They had no land due to Boone's bad investments.

C. A balanced diet contains proteins, which are composed of complex amino acids. There are 20 types of amino acids, comprising about 16 percent of the body weight in a lean individual. A body needs all 20 to be healthy. Amino acids can be divided into two groups: essential and nonessential. There are 9 essential amino acids. These are the proteins that the body cannot produce by itself, so a healthy individual must ingest them. The 11 nonessential amino acids, on the other hand, are produced by the body, so it is not necessary to ingest them. Proteins are described as being either high-quality or low-quality, depending on how many of the 9 essential amino acids the food contains. High-quality proteins, typically found in animal meats, are proteins that have ample amounts of the essential amino acids. Low-quality proteins are mainly plant proteins and usually lack one or more of the essential amino acids. Since people who follow a strict vegetarian diet are ingesting only low-quality proteins, they must make sure that their diets contain a variety of proteins, in order to ensure that what is lacking in one food is available in another. This process of selecting a variety of the essential proteins is called protein complementation. Since an insufficient amount of protein in the diet can be crippling, and prolonged absence of proteins can cause death, it is imperative that a vegetarian diet contains an ample amount of the essential proteins.

1. With what topic is this passage primarily concerned? (A) the 20 types of amino acids (B) high- and low-quality proteins 2. The word "lean" could be best replaced by (A) thin (B) fat (C) tall (D) short (C) the process of complementation (D) healthy diets for vegetarians

115

HUNG VUONG SUMMER C AMP-2010

3. The word "ample" in this passage is closest in meaning to which of the following? (A) meager (B) frequent (C) substantial (D) harmful

4. Which of the following would NOT be an example of a low-quality protein? (A) legumes (B) apples (C) grains (D) tuna

5. As used in this passage, which of the following words is closest in meaning to "strict"? (A) responsible (B) casual (C) harmonious (D) rigid

6. According to the passage, a vegetarian could die from insufficient protein ingestion if he or she (A) did not follow a varied and properly protein-complemented diet. (B) ate too many animal proteins, and could not digest them properly. (C) did not follow a diet in which nonessential proteins were ingested. (D) ate too many low-quality proteins. 7. Which of the following is closest in meaning to the word "crippling" as used in this passage? (A) discouraging (B) betraying (C) incapacitating (D) amazing

8. Which of the following words could best replace the word "prolonged"? (A) narrow (B) hollow (C) hard-hearted (D) extended

9. In line 21, the word "imperative" can be best replaced by (A) crucial (B) impossible (C) wonderful (D) satisfying

10. Which of the following best describes the author's tone in this passage? (A) forceful (B) light (C) casual (D) argumentative

116

HUNG VUONG SUMMER C AMP-2010

PART VI: WRITING (30 points) 1. Rewrite the following sentences by completing the given words without changing the meaning of the orginal sentence. (10 points) 1. Most of the students ignored what the teacher was saying. Few 2. She discovered eight new comets in the course of her work. Her work resulted . 3. I find it surprising that she didnt like her present. I would 4. This scheme is too risky for my liking. The risks 5. If you dont believe it, youll never be able to do it. Believe

2. Write a ne w sentence using the word given. (5 points) 1. The police arrived as the thieves were committing the crime. (RED-HANDED) 2. We dont seem to have much sugar left. (RUNNING) 3. He celebrated his birthday last Saturday. (PLACE) 4. They lost not only their money but their passport as well. (ADDITION) 5. Bad weather delayed the flight. (DUE)

3. Writing a paragraph (20 points) A balanced diet is one that helps maintain or improve health. Do you agree with this statement? Give specific reasons and examples to support your answers. You should write at least 150 words.

117

HUNG VUONG SUMMER C AMP-2010

TEST 8

PART I: VOCABULARY AND GRAMMAR 1. Choose the best words or phrases to fill in the blanks circle A, B, C or D. 1. As I was _____through the newspaper this morning, I came across a picture of an old army friend of mine. a. gazing b. glancing c. staring d. glimpsing 2. I wont _____these children making a noise in my house. a. allow b. made c. have d. permit 3. I think its _____your luck to drive without wearing a seatbelt. a. risking b. tempting c. proving d. pushing

4. It is _____impossible to tell the twins apart. a. virtually b. realistically c. extremely d. closely

5. I am not a serious investor, but I like to _____in the stock maket. a. splash b. splatter c. paddle d. dabble

6. On my birthday, my son presented me _____a beautiful bunch of violets. a. for b. with c. by d. of

7. They sent a sample of the pottery they had found to the laboratory so that a date could be _____on it. a. set b. placed c. fixed d. put 8. He says he doesnt like his new school because he looks like a _____out of water there. a. duck b. fish c. goose d. boat

9. Some animals are on the _____of becoming extinct. a. edge b. verge c. tip d. side

10. The opposition leaders _____the government for not taking action sooner. a. warned b. threatened c. criticized d. urged

11. When the stockmarket crashed, the traders were _____to beggary. a. collapsed b. reduced c. ruined d. inclined

12. As the _____owner of the property, I would be able to sign the application for credit. a. sole b. lone c. solitary d. lonely

13. My grand mother was partial _____a glass of sherry before dinner.

118

HUNG VUONG SUMMER C AMP-2010

a. for

b. of

c. with

d. to

14. _____everyone was at the village meeting, but the planning officer from the council was conspicuous by his absence. a. hardly b. nearly c. rarely d. completely

15. Many children who get into trouble in their early teens go o n to become _____offenders. a. persistent b. insistent c. consistent d. resistant

**Your answers: 1. ... 2. .... 3. 6. 4. 5. 7. 8. 9. 10. ..

11...12. .. 13. 14. ..15. ..

PART II: USE OF ENGLISH 1. Fill in the blank with a suitable preposition write your ans wers in numbered places. Many visitors to our area remark (16) many statues and monuments in our city. They serve (17) constant reminders of our history. They are also constant collectors (18) dirt and grime. That is where a special team of the city employees comes (19). This team is assigned the task (20) keeping the statues clean. Depending (21) size and composition, a statue can take one (22) three days to clean

thoroughly. It takes (23) a year to get (24) all the statues and then it is time to start again. However, the team members claim they never get bored. Each statue has its own personality, and they get to know each other (25) a friend.

**Your answers: 16. .. 17. .. .. 18. 24. 25. . 19. 20. 21. ..22. .. 23.

PART III: MAKING SENSE OF WHAT THE SPEAKER SAYS

119

HUNG VUONG SUMMER C AMP-2010

1. Read these dialognes and try to understand what the speaker means to say by circling A, B, C or D. 26. Woman: Why do you look so happy this morning? Man: I just came from my advisors office and found out that the college board has done away with the foreign language requirement for graduation. What do we learn from this conversation? a. The man doesnt have to study a foreign langnage. b. The man just received an A on his test. c. The mans adviser gave him some good advice. d. He doesnt have to take the final exam. 27. Man: Why are you buying that cheaper grade of meat, Clindy? Woman: Bob didnt get a cost of living raise, so we have to do without a few things now. What do we learn from this conversation? a. Their rent has been raised so they have to cut down on their grocery expenditure. b. They cant afford to eat meat c. The cheaper grade of meat comes without fat. d. They will have to cut down on expenses 28. Man: May I speak to Jason Daniels, please? Woman: Nobody by that name works here. What does the woman mean? a. Jason Daniel isnt home right now. b. The caller dialed the wrong number. c. Jason Daniels cant come to the phone right now. d. Jason Daniels doesnt want to speak to the caller. 29. John: How are you, Mary? I heard you were sick? Mary: They must have confused me with someone else. Ive never felt better. What does Mary mean? a. better b. sick c. fine d. tired 30. John: Can you help me? I havent done this before? Peter: Its easy. All you do is to put the worm on the hook, loosen the line and cast it. What is Peter showing John how to do? a. sail a boat b. hang clother c. eatch a horse d. fish

120

HUNG VUONG SUMMER C AMP-2010

**Your answers: 26. ..27. ..28.29. 30.

PART IV: Situational Use of Language Peter is looking for a place to live and has gone to an accommodation agency. Complete this dialogue by writing in numbered places. Peter: Im looking for a flat near the centre of town. Have you had anything suitable? Agent: (31) Peter: Oh, not more than $25 per week, I suppose. Agent: (32) Peter: Really? Well, I cant afford to pay that much. Agent: (33) Peter: well, I wanted something near the centre where my college is. If its far away I would have to spend a lot of time and money travelling. Agent: (34).. Peter: Id prefer a place to myself, but I suppose I dont mind sharing if that keeps the rent down. Agent: (35).

PART V: Gap filling: Fill in the blanks with suitable given words. (36) is a factor in education. (37) a global scale, national educational systems, intending to bring out the best in the next generation, (38) competitiveness among students by scholarships. Countries like Singapore and England have a special educational program (39) caters to special students, prompting charges of academic elitism. Upon receipt of their academic results, students tend to (40) their grades to see who is (41). For severe cases, the pressure to perform in some countries is (42) high that intellectually deficient students even suicide as consequence of (43) the exam, (44) Japan is a prime examp le. This has resulted (145) critical revaluation of examination as a whole by educationalists. 36. a. Compete 37. a. On 38. a. create 39. a. when 40. a. participate b. Competition b. With b. reise b. which b. observe c. Competitive c. At c. enconrage c. whole c. notice d. Competitor d. In d. found d. whom d. conpare

121

HUNG VUONG SUMMER C AMP-2010

41. a. the best 42. a. such 43. a. falling 44. a. so 45. a. in

b. the better b. so b. dropping b. but b. with

c. more better c. too c. failing c. because c. for

d. best d. much d. breaking d. and d. at

**Your answers: 36. .. 37. .. .. 38. 44. 45. 39. 40. 41. ..42. .. 43.

PART VI: Rewriting these sentences, using some suggestions. 46. The only way you can hope to succeed is by trial and error. Only 47. I am not a solitary person. Im sociable. Rather 48. I set foot in the house and at the same moment the fire alarm went off. Scarcely ........................................ 49. Its pointless to try and make him change his mind. Its. 50. I think you ought to have your coat dry-cleaned. I think

PART VII: READING COMPREHENSION 1. Read the passages and then choose the best answer. A, B, C, or D. A. It was the first photograph that I had ever seen, and it fascinated me. I can remember holding it at every angle in order to catch the flickering light from the oil lamp on the dresser. The man in the photograph was unsmiling, but his eyes were kind. I had never met him, but I felt line that I knew him. One evening when I was looking at the photograph, as I always did before I went to sleep, I noticed a shadow across the mans thin face. I moved the photograph so that the shadow lay perfectly around his hollow cheeks. How different he looked! That night I could not sleep, thinking about the letter that I would write. First, I

122

HUNG VUONG SUMMER C AMP-2010

would tell him that I was eleven years old, and that if he had a little girl my age, she could write to me instead of him. I knew that he was a very busy man. Then I would explain to him the real purpose of my letter. I would tell him how wonderful he looked with the shadow that I had seen across his photograph, and I would most carefully suggest that he grow whiskers. Four months later when I met him at the train station near my home in Westfield, New York, he was wearing a full beard. He was so much taller than I had imagined from my tiny photograph. Ladies and gentlemen, he said, I have no speech to make and no time to make it in. I appear before you that I may see you and that you may see me. Then he picked me right up and kissed me on both cheeks. The whiskers scratched. Do you think I look better, my little friend? he asked me. My name is Grace Be dell, and the man in the photograph was Abraham Lincoln. 51. What is the authors main purpose in the passage? a. To explain how Grace Be dell took a photograph of Abraham Lincoln. b. To explain why Abraham Lincoln wore a beard. c. To explain why the first photographs were significant in American life. d. To explain why Westfield is an important city. 52. The word fascinated in line 1 could best be replaced by ______. a. interested b. frightened c. confused d. d isgusted 53. The word flickering in line 2 is closest in meaning to ______. a. burning constantly brightly 54. The man in the photograph ______. a. was smiling kind 55. What did Grace Be dell do every night before she went to sleep? a. She wrote letter photograph. c. She made shadow figures on the wall. 56. The little could not sleep because she was ______. a. sick b. excited c. lonely d. sad d. She read stories. b. She looked at the b. had a beard c. had a round, fat face. d. looked b. burning unsteadily c. burning very dimly d. burning

57. Why did the little girl write the man a letter?

123

HUNG VUONG SUMMER C AMP-2010

a. She was lonely her. c. She wanted him to grow a beard. 58. The word it in line 13 refers to ______. a. time b. speech

b. She wanted his daughter to write to

d. She wanted him to visit her.

c. photograph

d. station.

59. From this passage, it may be inferred that ______. a. Grace Bedell was the only one at the train station when Lincoln stopped at Westfield b. There were many people waiting for Lincoln to arrive on the train. c. Lincoln made a long speech at the station in Westfield. d. Lincoln was offended by the letter. 60. Why did the author wait until the last line to reveal the identity of the man in the photograph? a. The author did not know it. b. The author wanted to make the reader feel foolish. c. The author wanted to build the interest and curiosity of the reader. d. The author was just a little girl. **Your answers: 51. .. 52. .. .. 53. 59. 60. . 54. 55. 56. ..57. .. 58.

B. Application for admission to the Graduate School at this university must be made on forms provided by the Director of Admissions. An applicant whose undergraduate work was done at another institution should request that two copies of undergraduate transcripts and degrees be sent directly to the Dean of the Graduate School. Both the application and the transcripts must be on file at least one month prior to the registration date, and must be accompanied by a non-refundable ten - dollar check or money order to cover the cost of processing the application. Students who have already been admitted to the Graduate School but were not enrolled during the previous semester should reapply for admission using a special short form available in the office of the Graduate School. It is not necessary for students w ho have previously been denied admission to resubmit transcripts; however, new application forms

124

HUNG VUONG SUMMER C AMP-2010

must accompany all requests for reconsideration. Applications should be submitted at least eight weeks in advance of the session in which the student wishes to enroll. Students whose applications are received after the deadline may be considered for admission as non- degree students, and may enroll for six credit hours. Non-degree status must be changed prior to the completion of the first semester of study however. An undergraduate student of this university who has senior status and is within ten credit hours of completing all requirements for graduation may register for graduate work with the recommendation of the chairperson of the department and the approval of the Dean of the Graduate School. 61. What is the authors main point? a. How to apply to the Graduate School c. How to register for graduate coursework graduation 62. Where would this passage most probably be found? a. In a university catalog b. In a travel folder c. In a newspaper d. In a textbook b. How to obtain senior status d. Ho w to make application for

63. According to this passage, where would a student secure app lication forms admission to the university? a. From the chairperson of the department Graduate School c. From the institution where the undergraduate work was done Admissions 64. Which of the following documents must be on file thirty days before the registration date? a. Two copies of recommendations from former professors b. A written approval of the Dean of the Graduate School c. One set of transcripts and an English proficiency score d. Two copies of undergraduate courses and grades, an application form, and an application fee 65. The author uses the word non-refundable in line 5 to refer to ______. a. process b. an application c. a check d. a date 66. The phrase in advance of in line 11 is closest in meaning to ______. a. into b. on either side of c. after the end of d. prior to d. From the Director of b. From the Dean of the

125

HUNG VUONG SUMMER C AMP-2010

67. The author makes all of the following observations about non- degree students EXCEPT______. a. they may be admitted after the deadline b. they may enroll for six credit hours c. they must change their status during the first semester d. they need not submit transcripts 68. The word status in line 13 could best be replaced by which of the following? a. information b. classification c. payment d. agreement

69. Students who have already been admitted to the Graduate School ______. a. never need to apply for readmission b. must reapply if they have not been registered at the university during the previous semester c. must reapply every semester d. must reapply when they are within ten credit hours of graduation 70. What special rule applies to undergraduate students? a. They may not register for graduate work b. They must pass an examination in order to register for graduate work c. They may receive special permission to register for graduate work d. They may register for graduate work at any time. Your answers: 61. .. 62. .. .. 63. 69. 70. . 64. 65. 66. ..67. .. 68.

C. While many nineteenth-century reformers hoped to bring about reform through education or by eliminating specific evils, some thinkers wanted to start over and remake society by founding ideal, cooperative communities. The United States seemed to them a spacious and unencumbered country where models of a perfect society could succeed. These communitarian thinkers hoped their success would lead to imitation, until communities free of crime, poverty, and other social ills would cover the land. A number of religious groups, notably the Shakers, practiced communal living, but the main impetus to found model communities came from non-religious, rationalistic thinkers.

126

HUNG VUONG SUMMER C AMP-2010

Among the communitarian philosophers, three of the most influential were Robert Owen, Charles Fourier, and John Humphrey Noyes. Owen, famous for his humanitarian policies as owner of several thriving textile mills in Scotland, believed that faulty environment was to blame for human problems and that these problems could be eliminated in a rationally planned sociery. In 1825 he put his principles into practice at New Harmony, Indiana. The community failed economically after a few years but not before achieving a number of social successes. Fourier, a commercial employee in France, never visited the United States. However, his theories of cooperative living influenced many Americans through the writings of Albert Brisbane, whose Social Destiny of Man explained Fourierism and its self- sufficient associations or phalanxes. One or more of these phalanxes was organized in every Northem state. The most famous were Red Bank, New Jersey, and Brook Farm, Massachusetts. An early member of the latter was the author Nathaniel Hawthorne. Noyes founded the most enduring and probably the oddest of the utopian communities, the Oneida Community of upstate New York. Needless to say, none of these experiments had any lasting effects on the patterns of American society. 71. The main topic of the passage is ______. a. nineteenth-century schools c. the philosophy of Fourierism b. American reformers d. model communities in the nineeenth century

72. Which of the following is Not given in the passage as one of the general goals of communitarian philosophers? a. To remake society c. To establish ideal communities b. To spread their ideas throughout the United States d. To create opportunities through education

73. The Shakers are mentioned in line 6 as an example of ______. a. a communal religious c. rationalistic thinkers b. radical reformers d. an influential group of writers

74. Which of the following is closest in meaning to the word impetus in line 6? a. Stimulus b. Commitment c. Drawback d. Foundation

75. Which of the following is closest in meaning to the word thriving in line 10? a. Prosperous b. Famous c. Failing d. Pioneering 76. The phalanxes described in the second paragraph were an idea originally conceived by ______. a. Albert Brisbane c. Charles Fourier b. Robert Owen d. John Humphrey Noyes

127

HUNG VUONG SUMMER C AMP-2010

77. Why does the author mention Nathaniel Hawthorne in line 18? a. He founded Brook Farm in Massachusetts. b. He was a critic of Charles Fourier c. He wrote a book that led to the establishment of model communities d. He was at one time a member of the Brook Farm community 78. Which of the following communities lasted longest? a. New Harmony c. Red Bank b. The Oneida Community d. Brook Farm

79. The word oddest in line 18 is closest in meaning to which of the following? a. Earliest b. Most independent c. Largest d. Most unusual

80. The author implies that, for readers, the conclusion of the passage is ______. a. obvious b. surprising c. absurd d. practical

*Your answers: 71. .. 72. .. .. 73. 75. 76. ..77. .. 79. 80. . 74. 78.

PART VIII: WRITING Describe the career you have chosen and the reasons for your choice. (about 350 words)

128

HUNG VUONG SUMMER C AMP-2010

TEST 9

PART I. VOCABULARY AND GRAMMAR 1. Choose the best answe r to complete each of the following sentences:

1. You are late again! Please, try to be ___________________ in future. A. accurate B. efficient C. punctual D. reliable 2. Id _________________ you didnt leave just at the moment. A. rather B. like C. better D. asked

3. From the hotel there is good ____________________ of the mountain. A. vision B. view C. sight D. picture

4. He lost the race because he ran out of petrol on the last ____________________. A. lap B. trip C. tour D. yourney 5. Its a long _______________________ from Tokyo to London. A. tour B. track C. flight D. travel

6. If she can make up such stories, she is certainly a very ___________________. A. imaginable B. imaginative C. imaginary D. imagining

7. Mozart was a very ____________________ musician, and had written several concertos by the age og 10. A. gifted B. endowed C. expert D. qualified 8. the aim of the councils latest campaign is to ________________the use of the new recycling facilities. A. project B. prosecute C. promote D. provide

9. While their mother is away at work, the children have to _____________ for themselves. A. look B. charge C. rely D. fend

10. Since they started campaigning foe their cause, many people have come out in ______. A. return B. sympathy C. condolence D. consent

11. The party was so good that it continued on into the ________________ hours. A. tardy B. little C. tiny D. early

12. The damage from the explosion ___________________ to over $50 million. A. amounted B. cost C. estimated D. added

13. The better ________________ you have of words, the better meaning you can express.

129

HUNG VUONG SUMMER C AMP-2010

A. demand

B. command

C. market

D. performance

14. Its distressing to see small children _______________________ in the street. A. pleading B. imploring C. begging D. entreating 15. Hes very amusing, and I like his ____________________ of humour. A. meaning B. feeling C. mind D. sense

PART II. USE OF ENGLISH 1. Make sentences with given words: 1. He/ leave/ house/ without/ say/ a word. ......................................................................................................................................... 2. Hardly/ I/ close/ eyes/ when/ dream/ terrible dreams. ......................................................................................................................................... 3. She/ no sooner/ agree/ marry/ him/ than/ began/ hate/ him. ......................................................................................................................................... 4. Tom/ always borrow/ friends/ and never/ pay/ back. ......................................................................................................................................... 5. The boys/ clever enough/ find/ solution/ problem/ quickly. ......................................................................................................................................... 6. In/ opinion/ violent films/ should not/ show/ television. ......................................................................................................................................... 7. mother/ still/ hospital/ but/ out/ danger/ now. ......................................................................................................................................... 8. Jim think/ it/ more practical/ plant tulips/ to plant beans. ......................................................................................................................................... 9. spaceship/ return/ earth/ by 4p.m/ next week. ......................................................................................................................................... 10. If/ population/ not control/ earth/ over crowded. ......................................................................................................................................... 2. Finish each of the following sentences in such a way that it means exactly the same as the sentence printed before it. 1. John has not had his hair cut for over six months. The last ........................................................................................................................... 2. She got down to writing the report as soon as she returned from her walk.

130

No sooner ....................................................................................................................... 3. Mr Tippers wife was very sorry she couldnt celebrate the New Year with her husband.

HUNG VUONG SUMMER C AMP-2010

Mrs Tipper greatly .......................................................................................................... 4. You can take any of the routes, it will still take you about an hour to get there. Whichever ...................................................................................................................... 5. She was so anxious while the results were read out. ( BREATH) She ..................................................................................................................................

3. Making sense of what the speaker say: 1. Peter: What would you like to eat, Edith? Edith: A meat sandwich. Peter: Jean? Would you like a meat sandwich or a cheese sandwich? Jean: A cheese sandwich, please, Peter. Peter: Good evening. Well have one meat sandwich and two cheese sandwiches. Edith: And three teas, please. Where are they? A. C. They are in Peters house. They are in Jeans house. B. They are in Ediths house. D. They are in a restaurant.

2. Hostess Anne: Hostess Alice:

Alice! Perhaps that passenger is a hijacker. Which passenger, Anne? That sad man with the camere? Hes wearing black slacksand a jacket.

Hostess Anne: No, that fat lady with the big black handbag in her lelf hand. You see, When she went into the lavatory she didnt have that handbag in her hand, and now shes ........................................ Fat lady: (Clapping her hands) EVERY STAND! Im a hijacker. And in this handbag I have a ........................... Which is the best topic? A. A bad flight. C. Bad hostesses B. A fat lady D. A bad hijacker

3.

Jimmy:

Excuse me!

Mrs Green: Can I help you?

131

HUNG VUONG SUMMER C AMP-2010

Jimmy:

Can I have a film for my camera?

Mrs Green: How many exposures? Jimmy: Ah! 40, please. What does exposures mean? A. How many pictures? C. How much does it cost? B. How many films? D. How many cameras?

4. Marry: Alice:

Have some ice-scream with strawberries. I would rather yogurt. Thanks anyway. B. She doesnt like ice-scream. D. She cant eat ice-sceam

What does Alice mean? A. She thanks Marry. C. She doesnt like yogurt

5. Tom: When is she going to marry? Tim: Not until shes thirty- five. Shes going to marry when shes ................ A. 34 years old. C. 36 years old B. until 35 years old D. not 35 years old

PART III. Use the following information to make up a dialogue between Hoa and Anna (a foreign student). Holiday/ Celebration Mid- Autumn Festival (15th day of the 8th lunar month) Its main purpose People celebrate the largest full moon in the year Its activities Children wear masks, parade in the street, have parties with special cakes and lots of fruits, etc. Hoa: Do you know that Mid-Autumn Festival is next month? Anna: What is Mid-Autumn Festival? Hoa: Its time when vietnamese (1).................................................................................. Anna: When (2)......................................................? Hoa: Well, (3)................................................................................................................... Anna: (4)................................................................................ at Mid-Autumn Festival?

132

HUNG VUONG SUMMER C AMP-2010

Hoa: (5)............................................................................................................................... .................................................................................................................................. Anna: Oh, that sounds really interesting.

PART IV. READING COMPREHENSION: 1.. Fill in each numbe red blank with one suitable word or phrase. In the western customs (1) _____ hands is the customary form of greeting, but in China a nod of the head or (2) _____ bow is sufficient. Hugging and kissing when greeting are uncommon. Business cards are often (2) _____ and yours should be printed in your own language and in Chinese. Also, it is more respectful to present your card or a gift or -any other article using (4) _____ hands. The Chinese are (5) _____ applauders. You may be greeted with group clapping, even by small children. When a person is applauded in this practice it is the custom for that person to return the applause or a "thank you." When walking in public places, direct eye (6) _____ .and staring is uncommon in the larger cities, especially in those areas accustomed to foreign visitors. (7) _____, in smaller communities, visitors may be the subject of much curiosity and therefore you may notice some stares. (8) _____ speaking, the Chinese are not a touch-oriented society, especially true for visitors. So, avoid (9) _____ or any prolonged form of body contact. Public displays of affection are very rare. On the other hand, you may note people of the same sex walking hand- in-hand, which is simply a gesture of friendship. Do not worry about a bit of pushing and shoving in stores or when groups board public buses or trains. In this case, (10) _____ are neither offered or expected. The Chinese will stand much closer than Westerners. 1. 2. 3. 4. 5. 6. 7. 8. 9. 10. a. taking a. small a. exchanged a. pair a. enthusiast a. contact a. Moreover a. Generally a. touch a. Contacts b. bit b. changed b. couple b. shaking c. grasping c. slight c. transferred c. double d. light d. converted d. both d. hugging

b. enthusiastic c. enthusiasm d. enthusiastically b. look b. Furthermore b. Successfully b. to touch b. Apologies c. stare c. However c. Fortunately c. touched c. Gestures d. watch d. Whatever d. Expectedly d. touching d. Saying goodbye

133

HUNG VUONG SUMMER C AMP-2010

2. Read the passage carefully and choose the correct ans wer. University Entrance Examination is very important in Vietnamese students. High school graduates have to take it and get high results to be admitted to universities. The pressure on the candidates remains very high despite the measures that have been taken to reduce the heat around these exams, since securing a place in a state university is considered a major step towards a successful career for young people, especially those from rural areas or disadvantaged families. In the year 2004, it was estimated that nearly 1 million Vietnamese students took the University Entrance Examination, but on average only 1 out of 5 candidates succeeded. Normally, candidates take 3 exam subjects, and each lasts 180 minutes for the fixed group of subjects they choose. There are 4 fixed groups of subjects: Group A: Mathematics, Physics, and Chemistry; Group B: Mathematics, Biology, and Chemistry; Group C: Literature, History, and Geography; Group D: Literature, Foreign Language, and Mathematics. In addition to universities, there are community colleges, art and technology institutes; professional secondary schools, and vocational schools which offer degrees or certificates from a- few-month to 2- year courses. According to Vietnam's Ministry of Education and Training, there are currently 23 non-public universities, accounting for 11% of the total number of universities. These non-public universities are currently training 119,464 students, or 11.7% of the total number of students. The government is planning to increase the number of no n-public universities to 30% by 2007. 1. University Entrance Examination in Vietnamese is very _______. b. stressful c. free d. easy

a. interesting 2.

The word those refers to _______. b. young people c. universities d. Examinations

a. exam subjects 3.

In 2004, the proportion of the students who got success in University Entrance

Examination was about _______ percent. a. 5 4. b. 10 c. 20 d. 50

Which sentence refers to the University Entrance Examination in Vietnam?

a. Students find it easy to get success in the University Entrance Examination. b. Math is compulsory in the University Entrance Examination. c. Students are not allowed to choose their exam subjects. d. There are four fixed groups of exam subjects for students to choose.

134

HUNG VUONG SUMMER C AMP-2010

5.

According to the passage, _______.

a. the Vietnamese government will close all non-public universities by next year. b. the Vietnamese government does not appreciate non-public universities c. the Vietnamese government encourages the establishing of non-public universities. d. Vietnamese students have no alternative to continue their higher study besides universities.

3. Fill in each blank with ONE suitable word. Modern advertisements contain hidden (1) ___ ____. Implicit in the advertisement showing the pretty girl in the new car, or the smiling children round the packet of washing powder is the message (2) ___ ____ if we buy the product, we also achieve success and happiness. It is a subtle approach since it seeks to (3) ___ ____ our secret dreams, and it is inescapable since adver- ising is ubiquitous. (4) ___ ____ street billboards and catchy jingles on television bombard us from all sides . They brainwash us into believing that we can realize our ambitions quickly and easily. On the other hand, (5) ___ ____ of advertising say that it is beneficial. Advertising is informative . Advertisements tell us about useful new products. They (6) ___ ____ our lives with color and music. They increase demand, (7)___ ____ industry and so keep prices down . Whether for or against (8) ___ ____, most people would agree that some kind of supervision body, appointed by the government or by the advertising industry (9) ___ ____, is necessary to maintain (10) ___ ____ of honesty and to discourage the more blatant types of misleading advertisements.

4.. Fill in each numbe red blank with one suitable word or phrase. Researchers in communication show that more feelings and intentions are (1) _______ and received nonverbally than verbally. Mehrabian and Wienerfollowing have stated that only 7% (2) _______ message is sent through words, with remaining 93% sent nonverbal (3) _______. Humans use nonverbal communication because: 1. Words have limitations: There are numerous areas where nonverbal communication is

more (4) _______ than verbal, especially when we explain the shape, directions, personalities which are expressed nonverbally. 2. Nonverbal signal are powerful: Nonverbal cues primarily express inner (5) _______

while verbal messages deal basically with outside world. 3. Nonverbal message are likely to be more genuine: because nonverbal behaviors

cannot be controlled as easily as spoken words.

135

HUNG VUONG SUMMER C AMP-2010

4.

Nonverbal signals can express feelings inappropriate to state: Social etiquette limits

what can be said, but nonverbal cues can communicate thoughts. 5. A separate communication channel is necessary to help send complex messages: A

speaker can add enormously to the complexity of the verbal message through simple nonverbal signals. 1. 2. 3. 4. 5. a. sent a. through a. thought a. effect a. feelings b. posted b. in b. expressions b. effective b. words c. mailed c. of c. gestures c. effectively c. shows d. thrown d. for d. postures d. Effectiveness d. sorrows

PART V. ESSAY WRITING: Why should students go to university? Use your own ideas to support your writing with examples and relevant reasons. You should write at least 120 words.

136

HUNG VUONG SUMMER C AMP-2010

TEST 10

PART I. VOCABULARY AND GRAMMAR 1. Choose the best answer (A, or B, C, D) to finish each of the following sentences. (1.5 points) 1. If you had taken my advice, you....................able to deal with the problem now. A. will be B. would be C. are D. would have been 2. Dont................. up yet, you will soon be able to play the trumpet well. A. give B. catch C. break D. turn

3. Your behaviour makes me.................................... A. anger B. angrily C. angry D. in anger

4. The nurse was on............................ in the hospital all night. A. work B. alarm C. duty D. service

5. She went............. a bad cold just before Christmas. A. down with B. in for C. over D. through

6. There was nothing special about his clothes......................... from his flowery tie. A. but Answers: 1........................................................... 2............................................................. 3............................................................. 4............................................................. 5............................................................. 6.............................................................. B. except C. other D. apart

2. Complete the conversation below by using the suggested words in parentheses. Write your ans wers in the space provided. (1.5 points) (Wite the full possible answers) Huong: What is your hobby, Lan? Lan: (1)............................................................................................................................... (stamps) Huong: Can you tell me how you collect them? Lan: (2)............................................................................................................................... ............................................................................................................................................. (buy/ from/ post office/ ask/ friend/ relatives/ them) Huong: How do you organize stamps?

137

HUNG VUONG SUMMER C AMP-2010

Lan: (3)............................................................................................................................... ............................................................................................................................................. (classify them/ categories/ animals, plants, landscape and people) Huong: Which stamps, local ones or foreign ones do you prefer? Lan: (4)............................................................................................................................... (prefer/ foreign ones/ local ones) Huong: Where do you keep them? Lan: (5)............................................................................................................................... (small/ album) Huong: Why do you collect stamps? Lan: (6)............................................................................................................................... ............................................................................................................................................. (broaden knowledge/ know more/ countries and people/ the world) 3. Give the correct form of the words in parentheses. (1.5 points) 1. Every week, there are two (fly) from Ha Noi to Nha Trang. 2. He felt sad because he was (hope) in love. 3. In (add) to doing the cleaning, I make the coffee. 4. He likes to work on (science) subjects. 5. Dolphins are (nature) curious animals. 6. Visitors said that the (hospitable) of the people in Vietnam was most unforgettable. Answers: 1........................................................... 2.............................................................. 3. ............................................................ 4............................................................. 5. ........................................................... 6.............................................................

4. Choose one word whose unde rlined part (A, or B, C, D) is pronounced differently from that of the others. (1.0 point) 1. A. reasonable 2. A. says 3. A. learned 4. A. data Answers: 1. ................ 2. ................ 3. ................ 4. ................ B. relative B. pays B. unemployed B. capable C. eraser C. ways C. walked C. magical D. iron D. lays D. injured D. investigation

138

HUNG VUONG SUMMER C AMP-2010

5.

Find and correct ten mistakes left (not including the example) in the following

passage. The first one has been done as an example. (2.5 points) Example: are make to are made to

As more things are make to be sold and more people have services to sell, advertising grows. Today it is one of the bigest businesses. Every year people spend billions of dollars on advertising. Advertising help sell more things to more people. This in turn makes it possible produce more things to sell. Sometimes it even helps make things cost less. In the beginning, for example, radios costed much more then they do today because they were turn out slowly and expensive by hand. Yet, advertising made more people want radios. When manufacturers began making them by the thousands, they found quickly and cheaper ways of do the job because advertising encourages us buy and produce more things. It is sometimes called the spark plug of the business world. Answers: 1. ..................................................... 2. ..................................................... 3. ..................................................... 4. ..................................................... 5. ..................................................... 6. ..................................................... 7. ..................................................... 8. ..................................................... 9. ..................................................... 10. ...................................................

6. Read the passage and fill in the blank with an appropriate word. (2.5 points) Oxford is one of the most beautiful cities of England, and (1)....................... university is the oldest in the country. Every year hundreds (2)................... young people come up to Oxford from school to spend three or four years at the university, and they never forget their time (3)..................... People (4)................... come from other countries like to visit Oxford when they are in England (5).................. it has many interesting buildings. (6) ..................... you go to Oxford, you will see the colleges, the old churches, and the crowds (7)....................... people, cars and bicycles in the busy High Street, the (8).................... important street in Oxford. If you want, you can take a boat on the river (9)..................... the sunshine. You will enjoy your visit to this (10)................. city very much. Answers: 1. ................................................... 2. .................................................... 6. .................................................... 7. .....................................................

139

HUNG VUONG SUMMER C AMP-2010

3. .................................................... 4. .................................................... 5. ....................................................

8. ..................................................... 9. ..................................................... 10. ...................................................

7. Read the passage and choose the best answer A, or B, C, D. (2.5 points) There are many issues over computers and the education of todays children. One concern that parents face now is how young is too young to start educating a child about computers? Parents and teachers alike have mixed opinions as to when it is appropriate and most beneficial to the child. Many preschool children learn to work with a mouse and play interactive games before they learn to write or learn their ABCs. It is true that computers can be used as very effective tools for learning, but at what age do you begin? Some parents believe that computer education should be much like a young child taking piano or hockey lessons. They feel that if you start a child early enough you might find a computer genius. This will help develop the childs knowledge quickly while they are still young. Parents who currently work using computers understand the importance of computer education and desire to give their children a head start. On the other hand, some parents are concerned that computers deprive children of their creativity and socialization skills. They argue that children become brainwashed and less socialized by being exposed to computers too early. 1. Teachers and parents are not sure about.............................. A. B. C. D. how much computer games children should play. the time when children learn ABC. the suitable time to teach computers to children. the relationship between computers and children.

2. All of these groups of people can be the readers of this article EXCEPT: A. Parents B. Linguistics C. Educators D. Children

3. Where in the passage does the author talk about the benefits of computer education? A. Paragraph 1 B. Paragraph 2 C. Paragraph 3 D. Paragraph 1 and 2

4. This article can be taken from........................ A. a textbook C. a science fiction 5. What can be the best title for this article? A. B. Computer education for children. The influence of computer to children. B. a fashion magazine D. an educational magazine

140

HUNG VUONG SUMMER C AMP-2010

C. D.

How can children benefit from computers? Science and children.

Answers: 1.................. 2. ............ 3. .................... 4. .................... 5. .............

8. Read the passage and choose the best ans wer A or B, C, D to fill in the blank. (1.0 point) Can you imagine what life would be like if there were no telephone? You could not call up your friends on the phone and talk to them. If a fire broke (1)in your house you could not call the fire department. If somebody was sick, you could not call a doctor. (2).our daily life we need to communicate with one another. We need do this mostly by speaking to other people and listening to (3).they have to say to us, and when you are close to them we can do this very easily. However, our voices will not travel very far even when we shout. The man who made this possible was Alexandre Graham Bell, a Scotsman, born in Edinburgh in 1847. Bell, a teacher of visible speech who later moved to Canada, (4).all his spare time experimenting. So enthusiastic was he in his research for a means for sending speech with electricity that he left much time for his day-to-day work and at one time it was almost penniless. 1. A. out 2. A. With 3. A. that 4. A. took Answers: 1. ................ 2. ................. 3. .................... 4. .................... B. in B. In B. this B. wasted C. up C. On C. what C. cost D. off D. At D. which D. spent

9. Finis h the second sentence in s uch a way that it has the same meaning as the original one. (1.0 point) 1. Why dont you put a better lock on the door, Daisy? said John. John suggested.................................................................................................................... 2. Their teacher is making them study hard. They are.............................................................................................................................. 3. I didnt steal the car, Bob said. Bob denied.......................................................................................................................... 4. Alan regretted asking Mary to lend him $ 20.

141

HUNG VUONG SUMMER C AMP-2010

Alan wished........................................................................................................................

10. Write a paragraph (about 120 - 150 words) about the advantages of the Internet in our daily life. (2.0 points)

11. Write a one-paragraph profile of Neil Armstrong. Use the information below. (1.5 points) Neil Armstrong: Born: 1949 to 1952: 1952: July 20th , 1969: 1971 to 1979: American astronaut, first person to walk on the moon August 5th , 1930, Ohio, The United States of America pilot in the United States Navy join the NASA astronaut program first human to step on the moons surface teach at the University of Cincinnati

1986: appoint vice chairman of the committee that investigated the Challenger shuttle disaster His famous singing: Thats one small step for man, one giant leap for mankind.

12. The tables below present the number of students and the ranges of marks that students in College of Foreign Languages get in the pre -test and the post-test of reading skill. (1.5 pts) Marks 5< 5-6 7-8 9 - 10 Total number of students Number of students 6 11 7 1 25

Table 1. The number of students getting each range of marks in the Pre-test

Marks

5<

5-6

7-8

9 - 10

Total number of students

Number of students

15

25

Table 2. The number of students getting each range of marks in the Post-test Describe the two tables and discuss the progress the students have made (about 120 150 words).

142

HUNG VUONG SUMMER C AMP-2010

PART III KEY TO CLASSROOM PRACTICE RESOURCES

143

HUNG VUONG SUMMER C AMP-2010

TEST 1

PART I: VOCABULARY AND GRAMMAR. 1D 2B 3A 4D 5D

PART II: USE OF ENGLISH Choose the word or phrase (A, B, C or D) that best fits the blank space in the following passage.

1. speaking 7. being 12. relation

2. acquired 8. as 13. take

3. common 9. run 14. given

4. at 10. had

5.react

6. sophisticated 11. mainly

15. Matter

PART III: MAKING SENSE OF WHAT THE SPEAKER SAYS. Choose the best answe r to each question. 1A 2A 3C 4B 5B

PART IV: READING COMPREHENSION: A. 1C 3C 2. It is the most advanced form of communication. 4A 5B 6D 7B 8 9B 10A

B. 1C 2B 3C 4D 5B 6D 7A 8B 9A 10. Admiration

C. 1. An internet company 2A 3D 4D 5C 6A 7A 8D 9A 10A

PART V: WRITING.

144

HUNG VUONG SUMMER C AMP-2010

I. Rewrite the sentences using the words given and other words so that they have similar meaning to the first. 1. I wish I had finished my homework last night . 2. As long as you keep calm , youll pass your driving test . 3. It often takes him 15 minutes to lead the buffalo to the field 4. By whom are the tigers fed at the zoo every day ? 5. Henry wishes that he hadnt bought a second hand-car . 6. No one who was there noticed anything strange . 7. I have never tasted (eaten) such a delicious cake in my life . 8. It is two years since she left university . 9. They (People) send newspapers to China by train . 10. I had my car serviced yesterday by the mechanic

II. Write a paragraph of 150-200 words discussing the uses of Internet in our daily life

TEST 2 PART I. PHONETICS 1. Choose the word whose underlined part is pronounced differently from the rest in each group. 1. b 2. d 3. b 4. c 5. b

2. Pick out the word whose stress is placed differently 6.d 7.c 8.a 9.c 10.c

PART II. VOCABULARY AND GRAMMAR 1. Circle the letter of the best answe r. 1b 2c 14d 3a 15a 4d 16d 5c 17b 6d 18b 7b 19a 8b 20a 9c 10b 11c 12c 13a

2. Use the prope r tense/form of verbs in pare ntheses. Write your ans wer in the space provided. 1. had seen 2. havent been 3. will be 4. could/ would have arrived

5. being appointed/ having been appointed 8. was wearing

6. had been killed 7. was seen 10. phoned

9. neednt have cooked

145

HUNG VUONG SUMMER C AMP-2010

3. Error correction: 1a 2b 3b 4d 5b

4. Complete the letter by writing one word in each gap. 1. too 6. Be 7. you 2. about 8. if/when 3. yourself 9. what 4. the 10. unless 11. all 5. that/which 12. but

5. Give the right form of the word in the capital letters 1. anxiety 6. unfashionable 2. novelist 7. entrance 3. inexpensive 8. unfurnished 4. qualifications 9. tasty 5. Injury

10. outgrown

6. Match the questions/statement (1-10) with the responses (a-j) below. 1c 2f 3g 4a 5j 6i 7e 8h 9b 10d

PART III. READING COMPREHENSION

1. Read the text below and decide which ans wer A, B, C or D best fits each space. 1B 2D 3A 4B 5C 6D 7A 8B 9C 10A 11A 12C 13D

14A 15B

2. Read the passage carefully and then choose the best answer to each sentence. 1c 2a 3d 4c 5a

3. Choose from the list the sentence which best summarizes each part of the article. There is an extra sentence which you do not need. 1E 2F 3A 4D 5C

PART IV. WRITING 1. For each of the sentences below, write a ne w sentence as similar as possible in meaning to the original sentence, but using the word given. This word must not be altered in any way. 1. 2. There was an agreement among the teachers to introduce the new methods. Dickens died without finishing his last novel.

146

HUNG VUONG SUMMER C AMP-2010

3. 4. 5.

The kidnappers let all the hostage leave yesterday. It came as a shock to John that he had failed his driving test. I dont approve of that sort of behavior.

2. Rewrite the following sentences so that the meaning stays the same 1. 2. 3. 4. 5. The governments immigration policy has been under review for some time now. They have been trying out/ testing a new flu vaccine since the beginning of the year. He is the most likely person to succeed in solving the problem. He met Jane, who was later become his wife, when he was at Cambridge. I am beginning to get more and more annoyed by his behavior.

TEST 3 PART I. VOCABULARY AND GRAMMAR 1. D 6. C 11. A 16. B 21. C 2. A 7. A 12. B 17. D 22. A 3. D 8. A 13. D 18. D 23. D 4. C 9. B 14. A 19. A 24. B 5. B 10. C 15. A 20. C 25. B

PART II. USE OF ENGLISH 1 26. as 31. less 36. which 27. after 32. and 37. set 28. in 33. called 38. everything 29. without 34. the 39. was 30. over 35. down 40. present

2. 41. different 46. expressive 42. cheerful 47. angular 43. successful 48. beginning 44. inspiration 49. unrealistic 45. African 50. symbolizing

PART III. MAKING SENSE OF WHAT THE SPEAKER SAYS

147

HUNG VUONG SUMMER C AMP-2010

51. D

52. C

53. A

54. D

55. D

Part IV 56. I cant work out the answer without calculator. 57. We were not taken in by his smooth manner. 58. City are bound to win. 59. Everyone who spoke to the victim is under suspicion. 60. We are on very good terms with our next-door neighbors. 61. It is really time I was going. 62. I paid all the money before collecting the goods, which was foolish. 63. Robert didnt know what he was going to do next. 64. Only when I checked the type, did I notice that it was flat. 65. Please inform the relevant authorities without delay.

PART V. READING COMPREHENSION 1. 66. T 71. C 67. T 72. C 68. F 73. C 69. F 74. B 70. T 75. A

Thang diem. 80 cau x 0,1 = 8d Bai viet (Question 3. COMPOSITION) 2 diem

TEST 4 I. 1. C 8. B 2. D 9. A 3. A 10. B 17. A 4. C 11.A 18. C 5. D 12. B 19. B 6. A 13. C 20. B 7. C 14. B

15. C 16. D II. 21. impoverished 25. continual 29. imaginative III.

22. Memorials 26. Non-verbal 30. Atypical

23. Electrocution 27. Requisitioned

24. Misdiagnosis 28. Empowered

148

HUNG VUONG SUMMER C AMP-2010

31. adapting -> adapted 33. less -> fewer 35. specialized -> special 37. climates -> climates 39. before -> ago IV. 41. was reading 45. will carry 49. was waiting V. 51. a 55. a 59. the VI. 61. C 62. B 67. D 68. A 73. A 74. C VII. 76. aware 80. changed 84. medical VIII. 77. When 81. Much 85. Solve 63. A 69. D 75. F 64. C 70. B 52. 56. The 60. 42. Had arrived 46. Had been living 50. Hadnt received

32. Millennium -> millenniums 34. Lack -> lacked 36. Year -> years 38. Abundant -> abundantly 40. From -> since

43. Have arrived

44.Is waiting 48. Left

47. Am really standing

53. A 57. The

54. 58. The

65. D 71. G

66. B 72. E

78. Diseases 82. Parts/ places

79. Cure 83. Live

86. You wont feel at home here until a few weeks have gone by./ until after a few weeks. 94. The accused was overcome with emotion. 95. Naturally, you will be penalized by having points deducted if you arrive late. 96. The floor strewn with the models clothes. 97. Our director wanted to be addressed ... 98. I cant say I enjoy having my writing torn to pieces in front of me. 99. A decision will have to be made --100. The possibility of the hotel needing new staff in the summer cant/ shouldnt be ruled out.

149

HUNG VUONG SUMMER C AMP-2010

TEST 5 PART I. VOCABULARY AND GRAMMAR. 1. D 7. D 12. B 2. B 8. C 13. C 3. C 9. D 14. B 4. A 10. A 15. A 5. C. 11. A \

PART II. USE OF ENGLISH 16. was 21. at 17. but 22. into 18. when 23. where 19. and 24. began/started 20. spent 25.composed/ written 26. which 27. it 28. from 29. that 30. although

PART III: SITUATIONAL USE OF ENGLISH 34. .. did it have any functions? 35. .. is it made of? 36. . did people wear it? 37. . it was made? 38. . When was it found? When did you find it? 39. . Of life did she have? 40. it like this

PART IV: READING COMPREHENSION

Passage 1 Passage 2

41. D 46. C 51. B

42. C 47. B 52. D 55. D 60. D 65. B

43. B 48. D 53. B 56. A 61. B

44. A 49. C

45. B 50. A

Passage 3

54. B 59. A 64. A

57. B 62. A

58. C 63. B

PART V: SENTENCE TRANSFORMATION

150

HUNG VUONG SUMMER C AMP-2010

A. Finish the second sentence in such a way that it means exactly the same as the sentence printed before it. 66. Gold costs twice as much as silver. 67. It was not until last year that John applied for that job. 68. No sooner had the farmer gone to bed than he felt asleep. 69. Ive yet to meet anyone who is more generous than Andrew. 70. I wish I hadnt asked her to stay. 71. Had it not been for the attendance of a famous film star, the party wouldnt have been to a success. 72. If only I could live in London for a year. 73. Due to bad weather the flight was delayed. 74. Someone has run off with my jewelry. 75. The fans were deeply disappointed by the result of the match. 76. They have a lot of hobbies and interests in common. 77. In terms of education his childhood years had been well spent. 78. Eating fewer sweets will save you having to visit the dentist so often. 79. Nobody offered to take part in the school concert. 80. Seeing Jim suddenly brought it home to me how much he must have suffered.

TEST 6

PART I : VOCABULARY AND GRAMMAR Question I: 10 points 1. C 6. D 2. A 7. B 3. C 8. C 4. C 9. B 5. C 10. A

Question II : 10 points 1. given 2. distinguished 3. had been sitting 6. will get 10. have got 4. had stolen 5. shouldnt have done 8. did he say Question III: 10 points 1. valuable 6. terrified 2. frightened 3. length 7. lowered 4. worldwide 5. tightens 10. heights 9. glancing

7. was returned

8. repeatedly 9. embarrassment

151

HUNG VUONG SUMMER C AMP-2010

Question IV: 10 points 1. to 2. of 3. been 4. as 5. did

6. for 7. to

9. was 11. it 12. for

PART II: READING Question I: 10 points 1. D 6. D 2. B 7. B 3. A 8. C 4. C 9. B 5. A 10. D

Question II: 10 points 1. B 6. B 2. D 7. B 3. D 8. A 4. A 9. B 5. C 10. B

Question III: 10 points 1. for 2. cause 6. lasts 7. food 3. energy 8. in 4. at 5. sun 10. expensive

9. however

PART III: WRITING Question I : 5 points 1. Only in a situation like this can children learn how to behave. 2. If you hadnt helped me, I couldnt have finished that difficult task. 3. I had all my crops wiped out completely by / in / during the storm. 4. There has been a considerable fall in the oil price in the past months. 5. Against everybodys / everyones / all expectation, she lost. Question II: 5 points 1. She worked very hard to make ends meet. 2. Its time you did your homework. 3. The exam was a piece of cake and I made no mistake. 4. I was the last to hear about the accident. 5. It makes no difference to me whether we have a meeting today or tomorrow. Question III: (writing a composition) 20 points

152

HUNG VUONG SUMMER C AMP-2010

TEST 7

PART I: VOCABULARY AND GRAMMAR (15 points) 1. 2. 3. 4. 5. C D C A A 6. 7. 8. 9. 10. A A D D A 11. 12. 13. 14. 15. A B B C C

PART II: USE OF ENGLISH (15 points) 1. Gulf 2. been 3. rig 4. sank 5. later 6. happened 7. leaking 8. charge 9. clean-up 10. depend 11. animals 12. progress 13. put 14. fire 15. which

PART III: MAKING SENSE OF WHAT THE SPEAKER SAYS (5 points) 1. C 2. B 3. A 4. D 5. A

PART IV: SITUATIONAL USE OF ENGLISH (5 points) Shop Assistant: Hi, (1) are you being helped? W: No, Im not. (2) Im interested in some scarves. Shop Assistant: All our scarves are in this section. What do you think of this one here? Its made of silk. B: Hm, it looks nice, but Id like to have something warm for the winter. A: Maybe you would like a heavy wool scarf. (3) How about this one? B: I think thats what I want. (4) How much is it? A: Itsseventy- five dollars plus tax. B: Its a little expensive. Do you think its possible to get a discount? A: Hm, since you like it so much, how about a 10 percent discount. Thats the best I can offer. B: Thats good. Could you wrap it up for me? A: Sure. Is there anything else I can get for you? B: No, that should be it. (5) Thank you.

153

PART V: READING COMPREHENSION ( 30 points) 1. 1. 2. 3. 4. 5. 2. 1. 2. 3. 4. 5. 3. 1. 2. 3. 4. 5. 6. 7. 8. 9. 10. Passage 1. A C A B C Passage 2. A C C A B Passage 3. D A C D D A C D A A 6. 7. 8. 9. 10. C A C B A 6. 7. 8. 9. 10. C B B D B

Summe r - 2010

PART VI: WRITING (30 points) 1. SENTENCE TRANSFORMATION (10 points) A. Finish each of the sentences in such a way that it means exactly the same as the sentence printed before it. (5 points) 1. Few students listened to/ took note of what the teacher was saying. 2. Her work resulted in the discovery of eight new comets. 3. I would have thought that she would have liked her present. 4. The risks of this scheme/ involved in this scheme are too great for my liking. 5. Believe in it or/ otherwise youll never be able to do it. B. Write a new sentence using the word given. (5 points) 1. The police arrived as the thieves were committing the crime. (RED-HANDED) The police caught the thieves red-handed. 2. We dont seem to have much sugar left. (RUNNING) We are running out of sugar/ the sugar is running out. 3. He celebrated his birthday last Saturday. (PLACE) His birthday celebration took place last Saturday. 4. They lost not only their money but their passport as well. (ADDITION) In addition to their money, they lost their passport as well. 5. Bad weather delayed the flight. (DUE) The flight was delayed due to bad weather.

2. WRITING A PARAGHRAPH (20 points) A balanced diet is one that helps maintain or improve health. Do you agree with this statement? Give specific reasons and examples to support your answers. You should write at least 150 words. 1. Content: 50% of total mark: a provision of all main ideas and details as appropriate

2. Language: 30% of total mark: a variety of vocabulary and structures appropriate to the level of English language gifted upper-secondary school students 3. Presentation: 20% of total mark: coherence, cohesion, and style appropriate to the level

of English language gifted upper-secondary school students.

155

TEST 8

I. VOCABULARY: (15 points) 1. B 11. B 2. C 3. D 4. A 13. D 5. D 14. B 6. B 15. A 7. D 8. B 9. B 10. C

12. A

II. USE OF ENGLISH: (10 points) 16. on 21. on 17. as 22. to 18. of 23. about 19. in 24. through 20. of 25. As

III. MAKING SENSE OF WHAT THE SPEAKER SAYS: (5 points) 26. A 27. D 28. B 29. C 30. D

IV. SITUATIONAL USE OF ENGLISH: (5 points) 31. Well see what we have. How much do you want to pay? 32. Well, Im afraid theres nothing under $40. 33. There may be something cheaper, a little way out of town. 34. You could get something center, but cheaper if you shared a room. 35. In that case, heres one that might suit you. Third floor room near the station at $ 30 a week.

V. GAP-FILLING: (10 points) 36. B 41. A 37. A 42. B 38. C 43. C 39. B 44. D 40. D 45. A

VI. REWRITING: (5 points) 46. Only by trial and error, can you hope to succeed. 47. Rather than being a solitary person, I am sociable. 48. Scarcely had I set foot in the house when the fire alarm went off. 49. Its a waste of time trying to make him chage his mind. 50. I think your coat needs dry-cleaning.

VII. READING COMPREHENSION: (20 points) *Passage 1: (10 points)

156

51. B 56. B

52. A 57. C

53. B 58. B

54. D 59. B

55. B 60. C

*Passage 2: (10 points) 61. A 66. D 62. A 67. D 63. D 68. B 64. D 69. B 65. C 70. C

*Passage 3: (10 points) 71. D 76. C 72. C 77. D 73. A 78. B 74. A 79. D 75. A 80. A

VIII. WRITING: (20 points) TOTAL: 100 points : 10 = 10

TEST 9

PART I. VOCABULARY AND GRAMMAR (15.0p) 1.C 2. A 3. B 4. A 5. C 6. B 7. A 8. C 9. D 10. B 11. D 12. A 13. B 14. C 15. D

PART II. USE OF ENGLISH I. Make sentences with given words. (10.0p) 1. He left the house with saying any/ a words. 2. Hardly had I closed my eyes when I dreamt terrible dreams. 3. She had no sooner agreed to marry him than she began to hate him. 4. He always borrows money from his friends and never pays it back. 5. The boys were clever enough to find the solution to the problem quickly. 6. In my opinion, violent films should not be shown on television. 7. My mother is stillin hospital but she is out of endanger now. 8. Jim think it is more practical to plant tulips than to plant beans. 9. The spaceship will have returned to the earth by 4p.m next week. 10. If the population is not controlled, the earth will be overcrowded. II. Finish each of the following sentences in such a way that it means exactly the same

157

as the sentence printed before it. (5.0p) 1. The last time John had his hair cut for over six months.No sooner had she returned from

her walk than she got down to writing the report. 2. 3. 4. Mrs Tipper greatly regretted not being able to celebrate the New Year with her husband. Whichever route you take, it will still take you about an hour to get there. She held her breathwhile the results were read out.

III. Making sense of what the speaker say: (5.0p) 1. D 2. D 3. A 4. B 5. B

PART III. SITUATIONAL USE OF ENGLISH: (5.0p) Use the following information to make up a dialogue between Hoa and Anna (a foreign student). 1. 2. 3. 4. 5. ........................... people celebrate the largest full moon in the year. ........................... is it exactly? ..............., this year its on the 3th of October. What do you usually do .........................................................? Well, children wear masks, parade in the street, have parties with special cakes and lots

of fruits, etc. PART IV. READING COMPREHENSION: I. Fill in each numbered blank with one suitable word or phrase. (10.0p) 1. b 6. a 2.c 7. c 3. a 8. a 4. d 9d 5. b 10. b

II. Read the passage carefully and choose the correct answer. (5.0p) 1. b 2. b 3. c 4. d 5. c

III. Fill in each blank with ONE suitable word. (10.0p) 1. message 2. that 3. exploit 4. huge 5. defenders 6. brighten 7. stimulate 8. advertising 9. itself 10. standard

IV. Fill in each numbered blank with one suitable word or phrase. (5.0p) 1. a 2. c 3. b 4. b 5. a

PART V. ESSAY WRITING: (30.0p)

158

159

Das könnte Ihnen auch gefallen